Вы находитесь на странице: 1из 15

JM Tuason and Co vs Bolanos

Plaintiffs complaint against defendant was to recover possession of a registered land. In


the complaint, the plaintiff is represented by its Managing Partner, Gregorio Araneta, Inc.,another
corporation. Defendant, in his answer, sets up prescription and title in himself thru"open,
continuous, exclusive and public and notorious possession under claim of ownership,adverse to the
entire world by defendant and his predecessors in interest" from "timeimmemorial". After trial, the
lower court rendered judgment for plaintiff, declaring defendant tobe without any right to the land
in question and ordering him to restore possession thereof toplaintiff and to pay the latter a monthly
rent. Defendant appealed directly to the Supreme Courtand contended, among others, that
Gregorio Araneta, Inc. can not act as managing partner for plaintiff on the theory that it is illegal for
two corporations to enter into a partnershipIssue:Whether or not a corporation may enter into a
joint venture with another corporation.Ruling:It is true that the complaint states that the plaintiff is
"represented herein by its ManagingPartner Gregorio Araneta, Inc.", another corporation, but there
is nothing against onecorporation being represented by another person, natural or juridical, in a suit
in court. Thecontention that Gregorio Araneta, Inc. cannot act as managing partner for plaintiff on
the theorythat it is illegal for two corporations to enter into a partnership is without merit, for the true
rule isthat
"though a corporation has no power to enter into a partnership, it may nevertheless enter into a
joint venture with another where the nature of that venture is in line with the
businessauthorized by its charter."
(Wyoming-Indiana Oil Gas Co. vs. Weston, 80 A. L. R., 1043, citing 2.Fletcher Cyc. of Corp.,
1082.). There is nothing in the record to indicate that the venture inwhich plaintiff is represented by
Gregorio Araneta, Inc. as "its managing partner" is not in linewith the corporate business of either of
them

J . M . T U A S O N & C O . , I N C . ,
r e p r e s e n t e d b y i t M a n a g i n g
P A R T N E R , G
R
E
G
O
R
I
A
A
R
A
N
E
T
A
,
I
N
C
.
,
plaintiff-appellee,-versusQUIRINO BOLAOS,
defendant-appellant.F A C T S : T h i s w a s a n a c t i o n t o r e c o v e r
p o s s e s s i o n o f a p a r c e l o f l a n d w h e r e t h e plaintiff was
represented by a corporation.I s s u e : W O N t h e c a s e s h o u l d b e
d i s m i s s e d o n t h e g r o u n d t h a t t h e c a s e w a s n o t brought
by the real property in interestHeld:No.

there is nothing to the c ontention that the pres ent action is


not brought bythe real party in interest, that is, by J. M. Tuason and Co.,
Inc. What the Ruleso f C o u r t r e q u i r e i s t h a t a n a c t i o n
be brought
in the name of,
b u t n o t necessarily
by
, the real party in interest. (Section 2, Rule 2.)

The complaint is signed by the law firm of Araneta and Araneta, "counsel
forplaintiff" and commences with the statement "comes now plaintiff,
throughits undersigned counsel. " It is true that the complaint
also states that theplaintiff is "repres ented herein by its
Managing Partner Gregorio Araneta, Inc.", another corporation

There is nothing against one corporation


b e i n g r e p r e s e n t e d b y another person, natural or juridical, in a
suit in court.

The c ontention that Gregori o Araneta Inc. cannot act as


managingp a r t n e r f o r p l a i n t i f f o n t h e
t h e o r y t h a t i t i s i l l e g a l f o r
t w o corporations to enter into a partnership is without merit, for the
truerule is that though a corporation has no power into a partnership, itmay
nevertheless enter into a joint venture with another where thenature of
that venture is in line with the business authorized b y
itscharter.
NOTE: Point of the case is about joint ventures
b e i n g t r e a t e d s e p a r a t e l y f r o m partnerships.
Tuason
does not exp lain wh y th ere wa s a differenc e in treatment
of corporat e involvem ent in partnerships as compared to that
wh en it come to j oint ventures
PNB vs CA
83 SCRA 237 Business Organization Corporation Law Corporations
Liability for Negligence

However, the Board of Directors wanted to raise the price to P3.00 per
picul. This Tuazon does not want hence he backed out from the agreement.
This resulted to Tapnio not being able to realize profit and at the same time
rendered her unable to pay her P2,000.00 crop loan which would have been
covered by her agreement with Tuazon.
Eventually, Tapnio was sued by her other creditors and Tapnio filed a third
party complaint against PNB where she alleged that her failure to pay her
debts was because of PNBs negligence and unreasonableness.
ISSUE: Whether or not Tapnio is correct.
HELD: Yes. In this type of transaction, time is of the essence considering
that Tapnios sugar quota for said year needs to be utilized ASAP otherwise
her allotment may be assigned to someone else, and if she cant use it, she
wont be able to export her crops. It is unreasonable for PNBs board of
directors to disallow the agreement between Tapnio and Tuazon because of
the mere difference of 0.20 in the agreed price rate. What makes it more
unreasonable is the fact that the P2.80 was recommended both by the bank
manager and PNBs VP yet it was disapproved by the board. Further, the
P2.80 per picul rate is the minimum allowable rate pursuant to prevailing
market trends that time. This unreasonable stand reflects PNBs lack of the
reasonable degree of care and vigilance in attending to the matter. PNB is
therefore negligent.
A corporation is civilly liable in the same manner as natural persons for
torts, because generally speaking, the rules governing the liability of a
principal or master for a tort committed by an agent or servant are the same
whether the principal or master be a natural person or a corporation, and
whether the servant or agent be a natural or artificial person. All of the
authorities agree that a principal or master is liable for every tort which it
expressly directs or authorizes, and this is just as true of a corporation as of
a natural person, a corporation is liable, therefore, whenever a tortious act is
committed by an officer or agent under express direction or authority from
the stockholders or members acting as a body, or, generally, from the
directors as the governing body.

Sia vs People
Facts: Jose Sia, president and GM of Metal Manufacturing Company of the
Phil., on behalf of said company, obtained delivery of 150 cold rolled steel
sheets valued at P71,023.60 under a trust receipt agreement. Said sheets
were consigned to the Continental Bank, under the express obligation on the
part of Sia of holding the sheets in trust and selling them and turning over
the proceeds to the bank. Sia, however, allegedly failed and refused to
return the sheets or account for the proceeds thereof if sold, converting it to
his own personal use and benefit. Continental Bank filed a complaint for
estafa against Sia. The trial court and CA ruled against Sia.
Issue: Whether or not Sia, acting as President of MMCP, may be held liable
for estafa
Held: Sia was acquitted. CA decision is reversed.
An officer of a corporation can be held criminally liable for acts or
omissions done in behalf of the corporation only where the law directly
requires the corporation to do an act in a given manner. In he absence of a
law making a corporate officer liable for a criminal offense committed by
the corporation, the existence of the criminal liability of he former may not
be said to be beyond doubt. Hence in the absence of an express provision of
law making Sia liable for the offense done by MMCP of which he is
President, as in fact there is no such provision under the Revised Penal
Code, Sia cannot be said to be liable for estafa.
Sia vs People
Lessons Applicable: Corporate Criminal Liability (Criminal Procedure)
FACTS:

Rita Tapnio owes PNB an amount of P2,000.00. The amount is secured by


her sugar crops about to be harvested including her export quota allocation
worth 1,000 piculs. The said export quota was later dealt by Tapnio to a
certain Jacobo Tuazon at P2.50 per picul or a total of P2,500. Since the
subject of the deal is mortgaged with PNB, the latter has to approve it. The
branch manager of PNB recommended that the price should be at P2.80 per
picul which was the prevailing minimum amount allowable. Tapnio and
Tuazon agreed to the said amount. And so the bank manager recommended
the agreement to the vice president of PNB. The vice president in turn
recommended it to the board of directors of PNB.

Sia was the President and General Manager of the Metal


Manufacturing of the Philippines Inc. (MEMAP)
He obtained 150 M/T Cold Rolled Sheets consigned to
Continental Bank and converted it into personal used instead of
selling it and turning over the proceeds
It resulted to a damage of 46,819 php, interest of 28,736.47 php
and forfeited deposit of 71,023.60 php

ISSUE: W/N Sia can be criminally charged.


HELD: NO. Acquit.

Sia did not act for and on behalf of MEMAP


For crimes committed by corp. officers criminally charged,
existence of criminal liability for which the petition is being
prosecuted must be clear and certain, here it may not be said to
be beyond reasonable doubt

Allegation v. evidence = strictly in harmony


The merchandise was manufactured before sold but although the
bank was aware of this, it was not in the trust agreement

Sia vs People
Petitioner, Jose O. Sia, was the president and general manager of MetalManufacturing of
the Philippines (MEMAP). He was convicted of estafa for hisfa ilure to
return th e cold rolled s teel sheets or accou nt for th e
proceeds of those which were sold, to Continental Bank, herein
complainant. Petitionercontend ed tha t he cann ot b e made liab le
for the crime cha rged as he only acted for and in behalf of MEMAP as its
president.ISSUE:Whether petitioner could be held liable for estafa.RULING: The Court ruled
in the negative. The case of People vs. Tan Boon Kong (54 Phil. 607)
provides for thegeneral principle that for crimes committed by a corporation, the
responsibleofficers thereof would personally bear the criminal liability as a corporation isan
artificia l p erson, an ab stract b eing. H owever, the C ourt ru led
tha t s uchprinciple is not applicable in this case because the act alleged to be a crime isnot
in the performance of an act directly ordained by law to be performed
bythe corporation. The act is impos ed by agreemen t of
parties, a s a p ractice observed in the usual pursuit of a business or a
commercial transaction. Theoffense may arise, if at all, from the peculiar terms and
condition agreed uponby the pa rties to th e tran saction, n ot by d irect
provis ion of the law. In th ea bsen ce of an exp ress provis ion
of law making the pet ition er liab le for th e criminal offense committed
by the corporation of which he is a president as inf a c t t h e r e i s n o s u c h
provisions in the Revised Penal Code under
w h i c h petitioner is being prosecuted, the existence of a criminal liability
on his partmay not be s aid to be beyond an y dou bt. In a ll
crim ina l p ros ecu tion s, th e existence of criminal liability for which the accused is
made answerable mustbe clear and certain. Further, the civil liability imposed
by the trust receipt ise x c l u s i v e l y o n t h e M e t a l C o m p a n y .
S p e a k i n g o f s u c h l i a b i l i t y a l o n e , t h e petitioner was never
intended to be equally liable as the corporation. Withoutbeing made so liable personally as the
corporation is, there would then be nobasis for holding him criminally liable, for any violation of
the trust receipt.

PEOPLE vs. CONCEPCION, 44 Phil. 126FACTS:


Venancio Concepcion, President of the Philippine National Bank and a
member of theBoard thereof, authorized an extension of credit in favor of
"Puno y Concepcion, S. en C. to themanager of the Apa rri branch
of the Phi lippine National Bank. "Puno y C oncepcion, S. en
C."was a co-partnership where Concepcion is a partner. Subsequently,
Concepcion was charged andfound guilt y in the Court of First
Instance of Caga yan with viola tion of s ection 35 of Act
No.2 7 4 7 . S e c t i o n 3 5 o f A c t N o . 2 7 4 7 p r o v i d e s
that the National Bank shall not, directly
o r indirectly, grant loans to any of the members of the board of directors
of the bank nor to agentsof the branch banks. Counsel for the
defense a rgu e that t he documents of record do not prove that
authority to make a loan was given, but only show the concession of a
credit. They averredthat the granting of a credit to the co partnership "Puno y Concepcion, S. en C." by
VenancioConcepcion, Presiden t of the Phil ippine National
Bank, is not a "loan" within the m eaning of section 35 of Act No.
2747.
ISSUE:
Whether or not the granting of a credit of P300,000
t o t h e c o - p a r t n e r s h i p " P u n o y Concepcion, S. en C." by
Venancio Concepcion, President of the Philippine Nation al
Bank, a "loan" within the meaning of section 35 of Act No. 2747.
HELD:
The Suprem e Court ru led in the affirmative. Th e "c redit" of
an individual means his ability to borrow money by virtue of the
confidence or trust reposed by a lender that he will paywhat he ma y
promis e. A "loan" means the d elivery b y on e part y and the
rec eipt by the other party of a given sum of money, upon an
agreement, express or implied, to repay the sum loaned,with or without
interest. The concession of a "credit" necessarily involves the granting of
"loans"up to the limit of the amount fixed in the "credit,
People v. Concepcion
G.R. No. 19190 (November 29, 1922)
FACTS:
Defendant authorized an extension of credit in favor of
Concepcion, a co-partnership. Defendants wife was a director of
this co-partnership. Defendant was found guilty of violating Sec. 35
of Act No. 2747 which says that The National Bank shall not, directly
or indirectly, grant loans to any of the members of the Board of
Directors of the bank nor to agents of the branch banks. This
Section was in effect in 1919 but was repealed in Act No. 2938
approved on January 30, 1921.
ISSUE:
W/N Defendant can be convicted of violating Sections of Act
No. 2747, which were repealed by Act No. 2938.

HELD:
In the interpretation and construction, the primary rule is to
ascertain and give effect to the intention of the Legislature. Section
49 in relation to Sec. 25 of Act No. 2747 provides a punishment for
any person who shall violate any provisions of the Act. Defendant
contends that the repeal of these Sections by Act No. 2938 has
served to take away basis for criminal prosecution. The Court holds
that where an act of the Legislature which penalizes an offense
repeals a former act which penalized the same offense, such
repeal does not have the effect of thereafter depriving the Courts
of jurisdiction to try, convict and sentence offenders charged with
violations of the old law
People v. Concepcion
G.R. No. 19190. November 29, 1922
FACTS:
By telegrams and a letter of confirmation to the manager of the Aparri
branch of the Philippine National Bank, Venancio Concepcion, President of
the Philippine National Bank, between April 10, 1919, and May 7, 1919,
authorized an extension of credit in favor of "Puno y Concepcion, S. en C."
in the amount of P300,000. "Puno y Concepcion, S. en C." was a
copartnership. Venancio Concepcion is a member of the board of directors
of this bank and was charged with a violation of Section 35 of Act No.
2747. Section 35 of Act No. 2747, provides that "The National Bank shall
not, directly or indirectly, grant loans to any of the members of the board of
directors of the bank nor to agents of the branch banks."
ISSUE:
Whether or not the granting of a credit to the copartnership "Puno y
Concepcion, S. en C." by Venancio Concepcion, President of the Philippine
National Bank, an "indirect loan" within the meaning of section 35 of Act
No. 2747 hence violative of said law.
HELD:
Yes. The prohibition against indirect loans is a recognition of the familiar
maxim that no man may serve two masters that where personal interest
clashes with fidelity to duty the latter almost always suffers. If, therefore, it
is shown that the husband is financially interested in the success or failure
of his wife's business venture, a loan to partnership of which the wife of a
director is a member, falls within the prohibition. A loan, therefore, to a
partnership of which the wife of a director of a bank is a member, is an
indirect loan to such director. The court is of the opinion that the statute
forbade the loan to his copartnership firm as well as to himself directly. The
loan was made indirectly to him through his firm.
Albert vs Univ Publishing Co
Mariano Albert entered into a contract with UniversityPu blish ing Co. ,
Inc. th rough Jose M. Aru ego, its Presiden t, w h e r e b y
U n i v e r s i t y w o u l d p a y p l a i n t i f f f o r t h e e x c l u s i v e right
to pub lish h is revised Commentaries on the Revis ed Penal Code.
The contract stipulated that failure to pay onei n s t a l l m e n t w o u l d
r e n d e r t h e r e s t o f t h e p a y m e n t s d u e . When University
failed to pay the second installment, Albertsu ed for collec tion an d
won. How ever, upon ex ecutio n, itwas found tha t Un ivers ity
was not registered with th e S EC.A l b e r t p e t i t i o n e d f o r a
w r i t o f e x e c u t i o n a g a i n s t J o s e M . Aruego as the rea l
defenda nt. Un iversity opposed , on the ground that Aruego was not
a party to the case.ISSUE:W O N A r u e g o c a n b e h e l d
p e r s o n a l l y l i a b l e t o t h e plaintiff.HELD: YES. The Supreme Court
found that Aruegor e p r e s e n t e d a n o n - e x i s t e n t e n t i t y a n d
induced not onlyAlbert but the court to believe in
such
representation.Aruego,
acting
as
r e p r e s e n t a t i v e o f s u c h n o n - e x i s t e n t principal, was the real
party to the contract sued upon, andthus a ssumed su ch privileges
and obligation s a nd b ecamepersonally liab le for the contra ct
entered in to or for oth er a c t s p e r f o r m e d a s s u c h
a g e n t . O n e w h o h a s i n d u c e d another to act upon
h i s w i l f u l m i s r e p r e s e n t a t i o n t h a t a corporation was duly
organized and existing under the law,cannot th ereafter set u p
agains t his victim the prin ciple of corporation by estoppel The
Supreme Court likewise held that the doctrine of c o r p o r a t i o n b y
e s t o p p e l c a n n o t b e s e t u p a g a i n s t A l b e r t since it wa s
Aruego w ho h ad ind uced h im to a ct upon h is (Aru ego's )
willfu l rep res en tation that U nivers ity ha d been duly organized
and was existing under the law
Mariano Alber v University Publishing Co., Inc.Bengzon, J.P. J. | 1965
Facts:
No less than three times have the parties here appealedto this Court.
In 1949, Albert sued University Publishing Co.(UPC). He alleged that UPC
was organized and existingunder PH laws and that thru its president Jose
Aruego(Aruego), they entered into a contract where UPC wouldpay him 30
thousand pesos for the exclusive right topublish his revised Commentaries
on the RPC and for his
share in previous sales of the books 1
st
edition; that UPCundertook to pay in 8 instalments of 3.5k and failure topay
one instalment would render the rest dueAlbert said UPC failed to pay the 2

nd
instalmentbut the latter countered that it was the former whoviolated their
contract by his failure to deliver themanuscript.Later, Albert died and Justo
Albert (hisadministrator) substituted him. The CFI then favouredJusto and
ordered UPC to pay him 23 thousand. The caseswent to SC which reduced
it to 15 thousand pesos.The CFI then ordered for the execution againstUPC
but at some point, Justo petitioned for a writ of execution against Aruego
(its president) because he andthe sheriff discov
ered that UPC wasnt registered in the
SEC. UPC countered by saying that Aruego was not aparty to the case so
the petition should be denied.
SC notes that UPC doesnt want Aruego
to be a
party to the case because if hes not a party, a separate
action will have to be filed by Justo which will result in himdealing with the
statute of limitations.The CFI denied the petition so Justo appealed.W/N
Aruego considered a party in the case. Yes.Ratio:Non-registration of UPC:Undisputed;on account of the non-registration it cannot beconsidered a corporation, not
even a corporation
de facto;
UPC then has no personality separate from Aruego, thuscannot be sued
independently;
Corporation-by-estoppel not invoked by UPC:Even if invoked, its not applicable;
Aruego represented a non-existent entity andinduced not only Justo but also
the court tobelieve such representation; (he signed thecontract as president
and stated the UPC wasregistered);One who has induced another to act upon his wilfulmisrepresentation that a
corporation was duly organizedand existing under the law, cannot thereafter
set upagainst his victim the principle of corporation by estoppel(Salvatiera
vs. Garlitos, 56 O.G. 3069);
Aruego is the real defendant:UPC who came to the court, but as said, it doesnot have independent
personaility; it is just aname;In reality, it was Aruego,
in reality, the one whoanswered and litigated, through his own law firm
ascounsel;
On Agency:A person acting or purporting to act on behalf of acorporation which has no
valid existence
assumes such privileges and obligations and becomes personally liable
for contracts entered into or for other actsperformed as such agent;
On due process question (since Aruego
wasnt named
in the case):Aruego was given his day in court;Parties to a suit are "persons who have a right to controlthe proceedings, to
make defense, to adduce and cross-examine witnesses, and to appeal from a
decision; inreality, it was Aruego who exercised these rights;
By due process of law we mean a law which hearsbefore it condemns;
which proceeds upon inquiry, andrenders judgment only after trial;
Summary:
The evidence is patently clear that Jose M. Aruego, acting as representative
of a non-existent principal,was the real party to the contract sued upon; that
he was theone who reaped the benefits resulting from it, so much so
thatpartial payments of the consideration were made by him; thathe violated
its terms, thereby precipitating the suit in question;and that in the litigation
he was the real defendant.
CASE REMANDED:
Lower court to hold supplementaryproceedings for the purpose of carrying
the judgment intoeffect against University Publishing Co., Inc. and/or Jose
M. Aruego
(because others might be liable to him forreimbursement or contribution)

Smith, Bell & Company (Ltd.), pet vs.Joaquin Natividad, Collector of


Customs of the port of Cebu, resp.
This is a petition for a writ of mandamus filed by the petitioner to compel
Natividad to issue acertificate of Philippine registry in favor of the former
for its motor vessel
Bato.
Facts:
Smith, Bell & Co., (Ltd.), is a corporation organized and existing under the
laws of the Philippine Islands.A majority of its stockholders are British
subjects. It is the owner of a motor vessel known as the Batobuilt for it in
the Philippine Islands in 1916, of more than fifteen tons gross The Bato was
brought toCebu in the present year for the purpose of transporting plaintiff's
merchandise between ports in theIslands. Application was made at Cebu,
the home port of the vessel, to the Collector of Customs for acertificate of
Philippine registry. The Collector refused to issue the certificate, giving as
his reason thatall the stockholders of Smith, Bell & Co., Ltd., were not
citizens either of the United States or of thePhilippine Islands. The instant

action is the result.Counsel argues that Act No. 2761 denies to Smith, Bell
& Co., Ltd., the equal protection of the lawsbecause it, in effect, prohibits
the corporation from owning vessels, and because classification
of corporations based on the citizenship of one or more of their stockholders
is capricious, and that ActNo. 2761 deprives the corporation of its property
without due process of law because by the passageof the law company was
automatically deprived of every beneficial attribute of ownership in the
Batoand left with the naked title to a boat it could not use .
Issue:
Whether the Government of the Philippine Islands, through its Legislature,
can deny the registry of vessel in its coastwise trade to corporations having
alien stockholders
Ruling:
Yes. Act No. 2761 provides:
Investigation into character of vessel
.

No application for a certificate of Philippine register shall beapproved until


the collector of customs is satisfied from an inspection of the vessel that it is
engaged ordestined to be engaged in legitimate trade and that it is of
domestic ownership as such ownership isdefined in section eleven hundred
and seventy-two of this Code.
Certificate of Philippine register
.

Upon registration of a vessel of domestic ownership, and of morethan


fifteen tons gross, a certificate of Philippine register shall be issued for it. If
the vessel is of domestic ownership and of fifteen tons gross or less, the
taking of the certificate of Philippine registershall be optional with the
owner.While Smith, Bell & Co. Ltd., a corporation having alien
stockholders, is entitled to the protectionafforded by the due-process of law
and equal protection of the laws clause of the Philippine Bill of Rights,
nevertheless, Act No. 2761 of the Philippine Legislature, in denying to
corporations such asSmith, Bell &. Co. Ltd., the right to register vessels in
the Philippines coastwise trade, does not belongto that vicious species of
class legislation which must always be condemned, but does fall
withinauthorized exceptions, notably, within the purview of the police
power, and so does not offend againstthe constitutional provision
Smith Bell Co vs Natividad
Smith, Bell & Co. is a corporation organized and existing under the laws of
the Philippine Islands;majority of the stockholders are British; owner of a
motor vessel known as the Batobrought toCebu for the purpose of
transporting Smith, Bell & Co.s merchandise between ports in theislands.application for registration was made at Cebu at the Collector of Customs--denied. Becausethey were not citizens of the US/Phils.-Act 2671, Sec.
1172. Certificate ofPhilippine Register.upon registration of a vessel of
domesticownership, and of more than 15 tons gross, a certificate of
Philippine register shall be issued forit. If the vessel is of domestic
ownership and of 15 tons gross or less, the taking of the certificateof
Philippine register shall be optional with the owner.-domestic ownership,
as used in this section, means ownership vested in the (a) citizens ornative
inhabitants of the Phil Islands; (b) citizens of the US residing in the Phil.
Islands; (c) anycorporation or company composed wholly of citizen of
Phils./US or both-plaintiffs contention: Act No. 2671 deprives the corp. of
its property without due process of lawbecause by the passage of the law,
the company was automatically deprived of every beneficialattribute of
ownership of the Bato and that they are left with a naked title they could
not use.Issue: WON Smith, Bell & Co. were denied of the due process of
law by the Phil. Legislature in itsenactment of Act 2761.Ruling: No.
(judgment affirmedplaintiff cant be granted registry.)RD: Act No. 2761,
in denying to corporations such as Smith, Bell & Co. Ltd., the right to
registervessels in the Phils. Coastwide trade, falls within the authorized
exceptions. Specifically withinthe purview of the police power. Literally
and absolutely, steamship lines are the arteries of thecommerce in the
Phils. If one be severed, the lifeblood of the nation is lost. If these
areprotected, security of the country and general welfare is sustained
Stonehill vs Diokno
Facts: Respondents issued, on different dates, 42 search warrants against
petitioners personally, and/or corporations for which they are officers
directing peace officers to search the persons of petitioners and premises of
their offices, warehouses and/or residences to search for personal properties
books of accounts, financial records, vouchers, correspondence, receipts,
ledgers, journals, portfolios, credit journals, typewriters, and other
documents showing all business transactions including disbursement
receipts, balance sheets and profit and loss statements and
Bobbins(cigarettes) as the subject of the offense for violations of Central
Bank Act, Tariff and Customs Laws, Internal Revenue Code, and Revised
Penal Code.
Upon effecting the search in the offices of the aforementioned corporations
and on the respective residences of the petitioners, there seized documents,
papers, money and other records. Petitioners then were subjected to
deportation proceedings and were constrained to question the legality of the
searches and seizures as well as the admissibility of those seized as
evidence against them.

On March 20, 1962, the SC issued a writ of preliminary injunction and


partially lifted the same on June 29, 1962 with respect to some documents
and papers.
Held:
a.

b.

c.

Search warrants issued were violative of the Constitution and the


Rules, thus, illegal or being general warrants. There is no
probable cause and warrant did not particularly specify the
things to be seized. The purpose of the requirement is to avoid
placing the sanctity of the domicile and the privacy of
communication and correspondence at the mercy of the whims,
caprice or passion of peace officers.
Document seized from an illegal search warrant is not
admissible in court as a fruit of a poisonous tee. However, they
could not be returned, except if warranted by the circumstances.
Petitioners were not the proper party to question the validity and
return of those taken from the corporations for which they acted
as officers as they are treated as personality different from that
of the corporation.

Stonehill vs Diokno
Stonehill et al and the corporation they form were alleged to have
committed acts in violation of Central Bank Laws, Tariff and Customs
Laws, Internal Revenue (Code) and Revised Penal Code. By the strength
of this allegation a search warrant was issued against their persons and their
corporation. The warrant provides authority to search the persons abovenamed and/or the premises of their offices, warehouses and/or residences,
and to seize and take possession of the following personal property to wit:
Books of accounts, financial records, vouchers, correspondence, receipts,
ledgers, journals, portfolios, credit journals, typewriters, and other
documents and/or papers showing all business transactions including
disbursements receipts, balance sheets and profit and loss statements and
Bobbins (cigarette wrappers).
The documents, papers, and things seized under the alleged authority of the
warrants in question may be split into (2) major groups, namely:

cause, for the same presupposes the introduction of competent proof that the
party against whom it is sought has performed particular acts, or committed
specific omissions, violating a given provision of our criminal laws. As a
matter of fact, the applications involved in this case do not allege any
specific acts performed by herein petitioners. It would be a legal heresy, of
the highest order, to convict anybody of a violation of Central Bank Laws,
Tariff and Customs Laws, Internal Revenue (Code) and Revised Penal
Code, as alleged in the aforementioned applications without
reference to any determinate provision of said laws or codes.
The grave violation of the Constitution made in the application for the
contested search warrants was compounded by the description therein made
of the effects to be searched for and seized, to wit:
Books of accounts, financial records, vouchers, journals, correspondence,
receipts, ledgers, portfolios, credit journals, typewriters, and other
documents and/or papers showing all business transactions including
disbursement receipts, balance sheets and related profit and loss
statements.
Thus, the warrants authorized the search for and seizure of records
pertaining to all business transactions of Stonehill et al, regardless of
whether the transactions were legal or illegal. The warrants sanctioned the
seizure of all records of Stonehill et al and the aforementioned corporations,
whatever their nature, thus openly contravening the explicit command of the
Bill of Rights that the things to be seized be particularly described as
well as tending to defeat its major objective: the elimination of general
warrants. The Moncado doctrine is likewise abandoned and the right of the
accused against a defective search warrant is emphasized.

Stonehill vs Diokno
1. Respondent (porsecution) made possible the issuance of 42 search
warrants against the petitioner and the corporation to search persons and
premises of several personal properties due to an alleged violation of
Central Bank Laws, Tariff and Custom Laws, Internal Revenue Code and
the Revised Penal Code of the Philippines. As a results, search and seizures
were conducted in the both the residence of the petitioner and in the
corporation's premises.

(b) those found seized in the residences of petitioners herein.

2.The petitioner contended that the search warrants are null and void as
their issuance violated the Constitution and the Rules of Court for being
general warrants. Thus,he filed a petition with the Supreme Court for
certiorari, prohibition, mandamus and injunction to prevent the seized
effects from being introduced as evidence in the deportation cases against
the petitioner. The court issued the writ only for those effects found in the
petitioner's residence.

Stonehill averred that the warrant is illegal for:

Issue: Whether or not the petitioner can validly assail the legality of the
search and seizure in both premises

(a) those found and seized in the offices of the aforementioned corporations
and

(1) they do not describe with particularity the documents, books and things
to be seized;
(2) cash money, not mentioned in the warrants, were actually seized;
(3) the warrants were issued to fish evidence against the aforementioned
petitioners in deportation cases filed against them;
(4) the searches and seizures were made in an illegal manner; and
(5) the documents, papers and cash money seized were not delivered to the
courts that issued the warrants, to be disposed of in accordance with law.
The prosecution counters, invoking the Moncado doctrine, that the defects
of said warrants, if any, were cured by petitioners consent; and (3) that, in
any event, the effects seized are admissible in evidence against them. In
short, the criminal cannot be set free just because the government blunders.
ISSUE: Whether or not the search warrant issued is valid.
HELD: The SC ruled in favor of Stonehill et al. The SC emphasized
however that Stonehill et al cannot assail the validity of the search warrant
issued against their corporation for Stonehill are not the proper party hence
has no cause of action. It should be raised by the officers or board members
of the corporation. The constitution protects the peoples right against
unreasonable search and seizure. It provides; (1) that no warrant shall issue
but upon probable cause, to be determined by the judge in the manner set
forth in said provision; and (2) that the warrant shall particularly describe
the things to be seized. In the case at bar, none of these are met. The warrant
was issued from mere allegation that Stonehill et al committed a violation
of Central Bank Laws, Tariff and Customs Laws, Internal Revenue (Code)
and Revised Penal Code. In other words, no specific offense had been
alleged in said applications. The averments thereof with respect to the
offense committed were abstract. As a consequence, it was impossible for
the judges who issued the warrants to have found the existence of probable

RULING: No, he can only assail the search conducted in the residences but
not those done in the corporation's premises. The petitioner has no cause of
action in the second situation since a corporation has a personality separate
and distinct from the personality of its officers or herein petitioner
regardless of the amount of shares of stock or interest of each in the said
corporation, and whatever office they hold therein. Only the party whose
rights has been impaired can validly object the legality of a seizure--a
purely personal right which cannot be exercised by a third party. The right
to object belongs to the corporation ( for the 1st group of documents,
papers, and things seized from the offices and the premises).
Stonehill vs Diokno
1. Respondent (porsecution) made possible the issuance of 42 search
warrants against the petitioner and the corporation to search persons and
premises of several personal properties due to an alleged violation of
Central Bank Laws, Tariff and Custom Laws, Internal Revenue Code and
the Revised Penal Code of the Philippines. As a results, search and seizures
were conducted in the both the residence of the petitioner and in the
corporation's premises.
2.The petitioner contended that the search warrants are null and void as
their issuance violated the Constitution and the Rules of Court for being
general warrants. Thus,he filed a petition with the Supreme Court for
certiorari, prohibition, mandamus and injunction to prevent the seized
effects from being introduced as evidence in the deportation cases against
the petitioner. The court issued the writ only for those effects found in the
petitioner's residence.
Issue: Whether or not the petitioner can validly assail the legality of the
search and seizure in both premises
RULING: No, he can only assail the search conducted in the residences but
not those done in the corporation's premises. The petitioner has no cause of
action in the second situation since a corporation has a personality separate
and distinct from the personality of its officers or herein petitioner
regardless of the amount of shares of stock or interest of each in the said
corporation, and whatever office they hold therein. Only the party whose
rights has been impaired can validly object the legality of a seizure--a
purely personal right which cannot be exercised by a third party. The right

to object belongs to the corporation ( for the 1st group of documents,


papers, and things seized from the offices and the premises).

ISSUE: W/N Benguet Consolidated, Inc. can ignore a court order because
of its by-laws

Tayag vs Benguet

HELD: NO. CFI Affirmed

26 SCRA 242 Business Organization Corporation Law Domicile of a


Corporation By Laws Must Yield To a Court Order Corporation is an
Artificial Being

In March 1960, Idonah Perkins died in New York. She left behind
properties here and abroad. One property she left behind were two stock
certificates covering 33,002 shares of stocks of the Benguet Consolidated,
Inc (BCI). Said stock certificates were in the possession of the Country
Trust Company of New York (CTC-NY). CTC-NY was the domiciliary
administrator of the estate of Perkins (obviously in the USA). Meanwhile,
in 1963, Renato Tayag was appointed as the ancillary administrator (of the
properties of Perkins she left behind in the Philippines).

Fear of contigent liability - obedience to a lawful order = valid


defense
Benguet Consolidated, Inc. is a Philippine corporation owing
full allegiance and subject to the unrestricted jurisdiction of local
courts
Assuming that a contrariety exists between the above by-law and
the command of a court decree, the latter is to be followed.
corporation is an artificial being created by operation of law...."It
owes its life to the state, its birth being purely dependent on its
will. Cannot ignore the source of its very existence

Tayag vs Benguet
A dispute arose between CTC-NY and Tayag as to who between them is
entitled to possess the stock certificates. A case ensued and eventually, the
trial court ordered CTC-NY to turn over the stock certificates to Tayag.
CTC-NY refused. Tayag then filed with the court a petition to have said
stock certificates be declared lost and to compel BCI to issue new stock
certificates in replacement thereof. The trial court granted Tayags petition.

BCI assailed said order as it averred that it cannot possibly issue new stock
certificates because the two stock certificates declared lost are not actually
lost; that the trial court as well Tayag acknowledged that the stock
certificates exists and that they are with CTC-NY; that according to BCIs
by laws, it can only issue new stock certificates, in lieu of lost, stolen, or
destroyed certificates of stocks, only after court of law has issued a final
and executory order as to who really owns a certificate of stock.
ISSUE: Whether or not the arguments of Benguet Consolidated, Inc. are
correct.
HELD: No. Benguet Consolidated is a corporation who owes its existence
to Philippine laws. It has been given rights and privileges under the law.
Corollary, it also has obligations under the law and one of those is to follow
valid legal court orders. It is not immune from judicial control because it is
domiciled here in the Philippines. BCI is a Philippine corporation owing
full allegiance and subject to the unrestricted jurisdiction of local courts. Its
shares of stock cannot therefore be considered in any wise as immune from
lawful court orders. Further, to allow BCIs opposition is to render the court
order against CTC-NY a mere scrap of paper. It will leave Tayag without
any remedy simply because CTC-NY, a foreign entity refuses to comply
with a valid court order. The final recourse then is for our local courts to
create a legal fiction such that the stock certificates in issue be declared lost
even though in reality they exist in the hands of CTC-NY. This is valid. As
held time and again, fictions which the law may rely upon in the pursuit of
legitimate ends have played an important part in its development.

PRIVATE INTERNATIONAL LAW: Situs of Shares of Stock:


domicile of the corporation
SUCCESSION: Ancillary Administration: The ancillary
administration is proper, whenever a person dies, leaving in a
country other than that of his last domicile, property to be
administered in the nature of assets of the deceased liable for his
individual debts or to be distributed among his heirs.
SUCCESSION: Probate: Probate court has authority to issue
the order enforcing the ancillary administrators right to the
stock certificates when the actual situs of the shares of stocks is
in the Philippines.

FACTS:
Idonah Slade Perkins, an American citizen who died in New York City, left
among others, two stock certificates issued by Benguet Consolidated, a
corporation domiciled in the Philippines. As ancillary administrator of
Perkins estate in the Philippines, Tayag now wants to take possession of
these stock certificates but County Trust Company of New York, the
domiciliary administrator, refused to part with them. Thus, the probate court
of the Philippines was forced to issue an order declaring the stock
certificates as lost and ordering Benguet Consolidated to issue new stock
certificates representing Perkins shares. Benguet Consolidated appealed the
order, arguing that the stock certificates are not lost as they are in existence
and currently in the possession of County Trust Company of New York.
ISSUE: Whether or not the order of the lower court is proper

HELD:
Further still, the argument invoked by BCI that it can only issue new stock
certificates in accordance with its bylaws is misplaced. It is worth noting
that CTC-NY did not appeal the order of the court it simply refused to
turn over the stock certificates hence ownership can be said to have been
settled in favor of estate of Perkins here. Also, assuming that there really is
a conflict between BCIs bylaws and the court order, what should prevail is
the lawful court order. It would be highly irregular if court orders would
yield to the bylaws of a corporation. Again, a corporation is not immune
from judicial orders.

Tayag vs Benguet
Lessons Applicable: Theory of Concession (Corporate Law)
FACTS:

March 27, 1960: Idonah Slade Perkins died in New York City
August 12, 1960: Prospero Sanidad instituted ancillary
administration proceedings appointing ancillary
administrator Lazaro A. Marquez later on substituted by Renato
D. Tayag
On January 27, 1964: CFI ordered domiciliary
administrator County Trust Company of New York to surrender
to the ancillary administrator in the Philippines 33,002 shares
of stock certificates owned by her in a Philippine corporation,
Benguet Consolidated, Inc., to satisfy the legitimate claims of
local creditors
When County Trust Company of New York refused the court
ordered Benguet Consolidated, Inc. to declare the stocks lost
and required it to issue new certificates in lieu thereof
Appeal was taken by Benguet Consolidated, Inc. alleging the
failure to comply with its by-laws setting forth the procedure to
be followed in case of a lost, stolen or destroyed so it cannot
issue new stock certs.

The

appeal

lacks

merit.

Tayag, as ancillary administrator, has the power to gain control and


possession of all assets of the decedent within the jurisdiction of the
Philippines
It is to be noted that the scope of the power of the ancillary administrator
was, in an earlier case, set forth by Justice Malcolm. Thus: "It is often
necessary to have more than one administration of an estate. When a person
dies intestate owning property in the country of his domicile as well as in a
foreign country, administration is had in both countries. That which is
granted in the jurisdiction of decedent's last domicile is termed the principal
administration, while any other administration is termed the ancillary
administration. The reason for the latter is because a grant of administration
does not ex proprio vigore have any effect beyond the limits of the country
in which it is granted. Hence, an administrator appointed in a foreign state
has no authority in the [Philippines]. The ancillary administration is proper,
whenever a person dies, leaving in a country other than that of his last
domicile, property to be administered in the nature of assets of the deceased
liable for his individual debts or to be distributed among his heirs."
Probate court has authority to issue the order enforcing the ancillary
administrators right to the stock certificates when the actual situs of the
shares
of
stocks
is
in
the
Philippines.
It would follow then that the authority of the probate court to require that
ancillary administrator's right to "the stock certificates covering the 33,002
shares ... standing in her name in the books of [appellant] Benguet
Consolidated, Inc...." be respected is equally beyond question. For appellant
is a Philippine corporation owing full allegiance and subject to the
unrestricted jurisdiction of local courts. Its shares of stock cannot therefore
be considered in any wise as immune from lawful court orders.
Our holding in Wells Fargo Bank and Union v. Collector of Internal
Revenue finds application. "In the instant case, the actual situs of the shares
of stock is in the Philippines, the corporation being domiciled [here]." To

the force of the above undeniable proposition, not even appellant is


insensible. It does not dispute it. Nor could it successfully do so even if it
were so minded.
Tayag vs Benguet
Idonah Slade Perkins, died in New York in 1960, left among others, two
stock certificates covering 33,002 shares of BenguetC o n s o l i d a t e d ,
the certificates being in the possession of the
County Trust Company of New York, the
d o m i c i l i a r y administrator of the estate of the deceased.A dispute
aros e bet ween the domiciary a dministrator in New York and
the ancillary administrator in the Philippines as to which of them
was entitled to the possession of the stock certificates in question. Anciallry
administrator wanted possessionof the shares so as to satisfy the legitimate
claims of local creditors.On January 27, 1964, the Court of First Instance of
Manila ordered the domiciliary administrator, County Trust Company,
to"produce and deposit" them with the ancillary administrator or with the
Clerk of Court. The domiciliary administrator did notcomply with the order,
and on February 11, 1964, the ancillary administrator petitioned the court to
"issue an order declaringthe certificate or certificates of stocks covering the
33,002 shares issued in the name of Idonah Slade Perkins by
BenguetConsolidated, Inc., be declared [or] considered as lost."T h e
order of the Lower Court is of the following tenor:
(1) considers as lost for all purposes in connection
w i t h t h e administration and liquidation of the Philippine
estate of Id onah Slade P erkins the stock certificates c overing
the 33,002 shares of stock standing in her name in the books of the
Benguet Consolidated, Inc., (2) orders said certificates cancelled,and (3)
directs said corporation to issue new certificates in lieu thereof, the same to
be delivered by said corporation to either the incumbent ancillary
administrator or to the Probate Division of this Court."Appeal t o the
order was mad e b y Bengu et Consolidated. Appellant oppos ed
the petition of the ancilla ry ad ministrator because the said
stock certificates a re in exist ence, they are toda y in the
possession of t he domiciliary a dministrator, the County Trust
Company, in New York, U.S.A...."
Issue/ Held: WON the appeal is meritorious.- NO. The order was called for
by the realities of the situation.Ratio:
The C ourt took into account the factual circumstances in
uphod ing the oder b y th e Lower Court that the shares of stock
be considered lost t for a ll purposes in connection with the
administration and liquidation of the Phi lippine estate
of Idonah Slade Perkins.1.Territorial scope of authority of administrator. It
is a "general rule universally recognized" that administration,
whether principal or ancillary, certainly "extends to the assets of a decedent
found within the state or country where it wasgranted," the corollary being
"that an administrator appointed in one state or country has no power over
property inanother state or country." Since the actual situs of the shares of
stock of a domestic corporation is in the Philippines,it should be
administered by the ancillary admisnitrator.
2.
Element of fiction of loss is necessary given the factual circumstances.
Since there is a refusal, persistentlyad hered to b y the domiciliary
administrator in New York, to deli ver the shares of stocks of
appellant corp oration owned by the decedent to the ancillary
administrator in the Philippines, there was nothing unreasonable or
arbitraryin considering them as lost and requiring the appellant to issue new
certificates in lieu thereof. Otherwise, to yield tothe stubborn refusal of the
domicillary administrator, the task incumbent under the law of the ancillary
administrator could not be discharged and his responsibility fulfilled.
3.
Lawful order of the court overrides the by-laws of Benguet Consolidated.
Benguet Consolidated stresses that in theevent of a contest or the
pendency of an action regarding own ership of such certificate
or c ertificates of st ock allegedly lost, stolen or destroyed, the issuance
of a new certificate or certificates would await the "final decision by[a]
court regarding the ownership [thereof]." SC held that Benguet
Consolidated's obedience to a lawful court order certainly constitutes a valid
defense, assuming that such apprehension of a possible court action against
it couldpossibly materialize.4 . A c o r p o r a t i o n i s n o t i m m u n e
f r o m j u d i c i a l a c t i o n . Definitions of Corporation:"...a
corporation is an artificial being creat ed by operation of
la w...."
It owes its life to the state, its birth being purely dependent on
its will. As Berle so aptly stated: "Classically, a corporation was conceived
as an artificial person, owing its existence through creation by a sovereign
power."
Berle, The Theory of Enterprise Entity, 47 Co. Law Rev. 343 (1907)."an
artificial being, invisible, intangible, and existing only in
contemplation of la w. " (Chief Justice Marshall,
Dartmouth College v. Woodward )"A corporation is not in fact and in reality a
person, but the law treats it as though it were a person by process of fiction,
or by regarding it as an artificial person distinct and separate from its
individual stockholders.... It owes its existence to law. It isan artificial
person created by law for certain specific purposes, the extent of whose
existence, powers and liberties is fixedby its charter." (Fletcher, Cyclopedia
Corporations )...a juristic person, resulting from an association of human
beings granted legal personality by the state, puts the matter neatly. (Pound
on Jurisprudence)There is thus a rejection of Gierke's

genossenchaft
theory, the basic theme of which to quote from Friedmann, "is the realityof
the group as a social and legal entity, independent of state
rec ognition and concession." A corporation as known
toPhilippine jurisprudence is a creature without any existence until it has
received the imprimatur of the state according to law.It is logically
inconceivable therefore that it will have rights and privileges of a higher
priority than that of its creator. More thanthat, it cannot legitimately refuse
to yield obedience to acts of its state organs, certainly not excluding the
judiciary, whenever called upon to do so.As a matter of fact, a
corporation once it com es into being, following Am erican la w
still of p ersuasive authorit y in our jurisdicti on, comes more
oft en within the ken of the judiciary than the other t wo
coordinate branches. It institutes the appropriate court action to
enforce its right. Correlatively, it is not immune from judicial control in
those instances, where aduty under the law as ascertained in an appropriate
legal proceeding is cast upon it.To assert that it can choose which court
order to follow and which to disregard is to confer upon it not autonomy
which maybe conceded but license which cannot be tolerated. It is to argue
that it may, when so minded, overrule the state, the sourceof its very
existence; it is to contend that what any of its governmental organs may
lawfully require could be ignored at will.So extravagant a claim cannot
possibly merit approval.
Umali vs CA
189 SCRA 529 Business Organization Corporation Law Piercing the
Veil of Corporate Fiction
Mauricia Castillo was the administratrix in charge over a parcel of land left
be Felipe Castillo. Said land was mortgaged to the Development Bank of
the Philippines and was about to be foreclosed but then Mauricias nephew,
Santiago Rivera, proposed that they convert the land into 4 subdivisions so
that they can raise the necessary money to avoid foreclosure. Mauricia
agreed. Rivera sought to develop said land through his company, Slobec
Realty Corporation (SRC), of which he was also the president. SRC then
contracted with Bormaheco, Inc. for the purchase of one tractor. Bormaheco
agreed to sell the tractor on an installment basis. At the same time, SRC
mortgaged said tractor to Bormaheco as security just in case SRC will
default. As additional security, Mauricia and other family members
executed a surety agreement whereby in case of default in paying said
tractor, the Insurance Corporation of the Philippines (ICP) shall pay the
balance. The surety bond agreement between Mauricia and ICP was secured
by Mauricias parcel of land (same land to be developed).
SRC defaulted in paying said tractor. Bormaheco foreclosed the tractor but
it wasnt enough hence ICP paid the deficiency. ICP then foreclosed the
property of Mauricia. ICP later sold said property to Philippine Machinery
Parts Manufacturing Corporation (PMPMC). PMPMC then demanded
Mauricia et al to vacate the premises of said property.
While all this was going on, Mauricia died. Her successor-administratrix,
Buenaflor Umali, questioned the foreclosure made by ICP. Umali alleged
that all the transactions are void and simulated hence they were defrauded;
that through Bormahecos machinations, Mauricia was fooled into entering
into a surety agreement with ICP; that Bormaheco even made the premium
payments to ICP for said surety bond; that the president of Bormaheco is a
director of PMPMC; that the counsel who assisted in all the transactions,
Atty. Martin De Guzman, was the legal counsel of ICP, Bormaheco, and
PMPMC.
ISSUE: Whether or not the veil of corporate fiction should be pierced.
HELD: No. There is no clear showing of fraud in this case. The mere fact
that Bormaheco paid said premium payments to ICP does not constitute
fraud per se. As it turned out, Bormaheco is an agent of ICP. SRC, through
Rivera, agreed that part of the payment of the mortgage shall be paid for the
insurance. Naturally, when Rivera was paying some portions of the
mortgage to Bormaheco, Bormaheco is applying some parts thereof for the
payment of the premium and this was agreed upon beforehand.
Further, piercing the veil of corporate fiction is not the proper remedy in
order that the foreclosure conducted by ICP be declared a nullity. The
nullity may be attacked directly without disregarding the separate identity
of the corporations involved. Further still, Umali et al are not enforcing a
claim against the individual members of the corporations. They are not
claiming said members to be liable. Umali et al are merely questioning the
validity of the foreclosure.
The veil of corporate fiction cant be pierced also by the simple reason that
the businesses of two or more corporations are interrelated, absent sufficient
showing that the corporate entity was purposely used as a shield to defraud
creditors and third persons of their rights. In this case, there is no
justification for disregarding their separate personalities.

Claparols vs CIR

A petition for certiorari to set aside the order of respondent Court of


Industrial Relations dated May 30, 1969 directing petitioners to pay
back wages and bonuses to private respondents as well as its
resolution of July 5, 1969 denying the motion for reconsideration of
said order in Case No. 32-ULP-Iloilo entitled "Allied Workers'
Association, et. al., versus Eduardo Claparols, et. al.."
It appears that on August 6, 1957, a complaint for unfair labor
practice was filed by herein private respondent Allied Workers'
Association, respondent Demetrio Garlitos and ten (10) respondent
workers against herein petitioners on account of the dismissal of
respondent workers from petitioner Claparols Steel and Nail Plant.
On September 16, 1963, respondent Court rendered its decision
finding "Mr. Claparols guilty of union busting and" of having
"dismissed said complainants because of their union activities," and
ordering respondents "(1) To cease and desist from committing
unfair labor practices against their employees and laborers; (2) To
reinstate said complainants to their former or equivalent jobs, as
soon as possible, with back wages from the date of their dismissal
up to their actual reinstatement" (p. 12, Decision; p. 27, rec.).

consideration of this Honorable Court (p. 2,


Report of Examiner, p. 29, rec.).
On January 23, 1965, petitioners filed an opposition alleging that
under the circumstances presently engulfing the company, petitioner
Claparols could not personally reinstate respondent workers; that
assuming the workers are entitled to back wages, the same should
only be limited to three months pursuant to the court ruling in the
case of Sta. Cecilia Sawmills vs. CIR (L-19273-74, February 20,
1964); and that since Claparols Steel Corporation ceased to operate
on December 7, 1962, re-employment of respondent workers cannot
go beyond December 7, 1962.
A reply to petitioner's opposition was filed by respondent workers,
alleging among others, that Claparols Steel and Nail Plant and
Claparols Steel and Nail Corporation are one and the same
corporation controlled by petitioner Claparols, with the latter
corporation succeeding the former.
On November 28, 1966, after conducting a series of hearings on the
report of the examiner, respondent Court issued an order, the
dispositive portion of which reads:

A motion to reconsider the above decision was filed by herein


petitioners, which respondent Court, sitting en banc, denied in a
resolution dated January 27, 1964.

WHEREFORE, the Report of the. Examiner filed


on January 15, 1965, is hereby approved
subject to the foregoing findings and
dispositions. Consequently, the Corporation
Auditing Examiner is directed to recompute the
back wages of complainants Demetrio Garlitos
and Alfredo Ongsuco on the basis of P200.00
and P270.00 a month, respectively; to compute
those of complainant Ignacio Quioyo as
aforesaid; to compute the deductible earnings of
complainants Ongsuco, Jorge Semillano and
Garlitos, as found in the body of this order; and
to compute the bonuses of each and every
complainant, except Honorato Quioyo.
Thereafter, as soon as possible, the Examiner
should submit a report in compliance herewith of
the Court's further disposition (p. 24, Brief for
Respondents, p. 113, rec.).

On March 30, 1964, counsel for herein respondent workers


(complainants in the ULP case) filed a motion for execution of
respondent Court's September 16, 1963 decision.
On May 14, 1964, respondent Court, in its order of September 16,
1963, granted execution and directed herein petitioners
to reinstate the above complainants to their
former or equivalent jobs within five (5) days
after receipt of a copy of this order. In order to
implement the award of back wages, the Chief
of the Examining Division or any of his
assistants is hereby directed to proceed to the
office of the respondents at Matab-ang, Talisay,
Negros Occidental, and examine its payrolls and
other pertinent records and compute the back
wages of the complainants in accordance with
the decision dated September 16, 1963, and,
upon termination, to submit his report as soon
as possible for further disposition (p. 7, Brief for
Respondents, p. 113, rec.).
which was reiterated by respondent Court in a subsequent order
dated November 10, 1964 (pp. 7-8, Brief for Respondents, p. 113,
rec.).
On December 14, 1964, respondent workers were accompanied by
the Chief of Police of Talisay, Negros Occidental to the compound of
herein petitioner company to report for reinstatement per order of the
court. Respondent workers were, however, refused reinstatement by
company accountant Francisco Cusi for he had no order from plant
owner Eduardo Claparols nor from his lawyer Atty. Plaridel Katalbas,
to reinstate respondent workers.
Again, on December 15, 1964, respondent workers were
accompanied by a police officer to the company compound, but then,
they were again refused reinstatement by Cusi on the same ground.

On December 7, 1966, a motion for reconsideration was filed by


petitioner, assailing respondent Court's ruling that (1) the ruling in the
case of Sta. Cecilia Sawmills Inc. CIR, et. al, does not apply in the
case at bar; and (2) that bonus should be included in the recoverable
wages.
On December 14, 1966, a counter-opposition was filed by private
respondents alleging that petitioners' motion for reconsideration was
pro forma, it not making express reference to the testimony or
documentary evidence or to the provision of law alleged to be
contrary to such findings or conclusions of respondent Court.
On February 8, 1967, respondent Court of Industrial Relations
dismissed petitioners' motion for reconsideration for being pro forma.
Whereupon, petitioners filed a petition for certiorari with this COURT
in G.R. No. L-27272 to set aside the November 28, 1966 order of
respondent Court, as well as its February 8, 1967 resolution.
Petitioners assigned therein as errors of law the very same
assignment of errors it raises in the present case, to wit:
I

On January 15, 1965, the CIR Chief Examiner Submitted his report
containing three computations, to wit:

THE RESPONDENT COURT ERRED AND/OR


ACTED WITH GRAVE ABUSE OF
DISCRETION, AMOUNTING TO LACK OF
JURISDICTION, IN HOLDING IN THE ORDER
UNDER REVIEW THAT BONUSES SHOULD
BE PAID TO THE RESPONDENT WORKERS
DESPITE THE FACT THAT THE SAME WAS
NOT ADJUDICATED IN ITS ORIGINAL
DECISION.

The first computation covers the period February


1, 1957 to October 31, 1964. The second is up
to and including December 7, 1962, when the
corporation stopped operations, while the third is
only up to June 30, 1957 when the Claparols
Steel and Nail Plant ceased to operate (Annex
B, Petition for Review on Certiorari, p. 14, Brief
for appellees, p. 113, rec.).
II
with the explanation that:
6. Since the records of the Claparols Steel
Corporation show that it was established on July
1, 1957 succeeding the Claparols Steel and Nail
Plant which ceased operations on June 30,
1957, and that the Claparols Steel Corporation
stopped operations on December 7, 1962, three
(3) computations are presented herein for the

THE RESPONDENT COURT ERRED AND/OR


ACTED WITH GRAVE ABUSE OF
DISCRETION, AMOUNTING TO LACK OF
JURISDICTION, IN NOT APPLYING THE
DOCTRINE LAID DOWN BY THIS
HONORABLE TRIBUNAL IN THE CASE OF
"STA. CECILIA SAWMILLS, INC. VS. C.I.R., ET.
AL.," G.R. No.

L-19273-74, PROMULGATED ON FEBRUARY


29, 1964 (pp. 10-11, rec.).
On April 27, 1967, the Supreme Court denied petitioners' petition for
certiorari (p. 77, rec. of L-27272), which was reiterated on May 19,
1967 (p. 27, Respondent's Brief, p. 113, rec.; p. 81, rec. of L-27272).
On May 3, 1967, private respondents moved to have the workers'
back wages properly recomputed. A motion to the same end was
reiterated by private respondents on June 14, 1967.
On July 13, 1967, respondent Court directed a recomputation of the
back wages of respondent workers in accordance with its order
dated November 28, 1966. The said order in part reads:
WHEREFORE, the Chief Auditing Examiner of
the Court or any of his assistants, is hereby
directed to recompute the back wages of the
workers involved in this case in accordance with
the Order of November 28, 1966 within 20 days
from receipt of a copy of this Order (p. 28, Brief
for Respondents, p. 113, rec.).
Then on March 21, 1968, the Chief Examiner came out with his
report, the disputed portion of which (regarding bonuses) reads:
xxx xxx xxx
4. The yearly bonuses of the employees and
laborers of respondent corporation are given on
the following basis:

On May 4, 1968, a rejoinder to petitioners' opposition was filed by


private respondents, alleging among others "that the grounds of
petitioners' opposition were the same grounds raised by them before
and passed upon by respondent Court and this Honorable Tribunal;
that this order of November 28, 1966 which passed upon these
issues became final and executory on June 3, 1967 from the
Honorable Supreme Court. (Order of respondent Court dated July
13, 1967). [p. 32, Brief for Respondents, p. 113, rec.].
On July 26, 1968, private respondents filed their motion for approval
of the Report of the Examiner submitted on March 21, 1968, alleging,
among others, that petitioners, in their opposition, did not actually
dispute the data elicited by the Chief Examiner but rather harped on
grounds which, as already stated, had already been turned down by
the Supreme Court.
On October 19, 1968, herein private respondents filed their
"Constancia", submitting the case for resolution of respondent Court
of Industrial Relations.
On May 30, 1969, respondent Court issued an order, subject of the
present appeal, the dispositive portion of which reads:
WHEREFORE, there being no proof offered to
substantiate respondent Eduardo Claparols'
opposition, the Examiner's Report should be,
and it is hereby, APPROVED. Consequently,
pursuant to the decision dated September 16,
1963, respondent ... (petitioners herein) are
hereby directed to pay the respective back
wages and bonuses of the complainants
(respondents herein) ... (p. 35, Brief for
Respondents; p. 113, rec.; emphasis
supplied).1wph1.t

Basic Additional:
a. For every dependent 1%
of monthly salary
b. For every dependent in
elementary grade 2% of
monthly salary
c. For every dependent in
high school 3% of monthly
salary
d. For every dependent in
college 5% of monthly
salary

On June 7, 1969, petitioners filed a motion for reconsideration on


practically the same grounds previously raised by them.
On June 30, 1969, respondents filed an opposition to petitioners'
motion for reconsideration, with the following allegations:
1. The issues raised, namely, whether bonuses
should be included in the award for back wages
had already been resolved by respondent court
in its orders dated November 28, 1966, and
December 7, 1966, and in the Resolution of the
Honorable Supreme Court in G.R. No. L-27272
dated April 26, 1967 and May 19, 1967, and the
same is already a settled and final issue.
2. Petitioners' motion for reconsideration is
merely a rehash of previous arguments, effete
and unrejuvenated, pro forma, and intended
merely to delay the proceedings.

xxx xxx xxx


7. The computed ... bonuses after deducting the
earnings elsewhere of Messrs. Ongsuco,
Garlitos and Semillano are as follows:
Name x x x Bonuses x x x
1. Alfredo Ongsuco
P1,620.00
2. Demetrio Garlitos
1,200.00
3. Ignacio Quioyo 455.23
4. Aser Abancio 461.00
5. Ludovico Belopeos
752.05
6. Salvador Doroteo 714.70
7. Rosendo Espinosa
1,075.40
8. Gaudencio Quioyo
1,167.92
9. Jorge Semillano 1,212.08
10. Maximo Quioyo 449.41
Total P9,107.79
(Pp. 30-31, Respondent's Brief, p. 113, rec.)
On April 16, 1968, petitioners filed their opposition to the report of the
Examiner dated March 21, 1968 on grounds already rejected by
respondent Court in its order dated November 28, 1966, and by the
Supreme Court also in its ruling in G.R. No. L-27272.

As correctly contended by private respondents, the present petition


is barred by Our resolutions of April 26, 1967 and May 19, 1967 in
G.R. No. L-27272 (Eduardo Claparols, et. al. vs. CIR, et. al.) [pp. 7783, rec. of L- 27272], dismissing said case, wherein said petitioners
invoked the applicability of the doctrine in Sta. Cecilia Sawmills, Inc.
vs. CIR, et. al. (L-19273-74, Feb. 29, 1964, 10 SCRA 433) and
impugned the illegality of the order of respondent Court dated
November 28, 1966 directing the computation and payment of the
bonuses, aside from back wages on the ground that these bonuses
were not included in the decision of September 16, 1963, which had
long become final.
The aforesaid resolutions in G.R. No. L-27272 constitute the law of
the instant case, wherein herein petitioners raised again practically
the same issues invoked in the abovementioned case. The denial of
the petition in G.R. No. L-27272 suffices to warrant the denial of the
present petition; and We need not go any further.
However, without lending a sympathetic ear to the obvious desire of
herein petitioners of this Court to re-examine which would be an
exercise in futility the final ruling in G.R. No. L-27272, which as
above-stated is the law of the instant case, but solely to remind
herein petitioners, We reiterate the governing principles.
WE uniformly held that "a bonus is not a demandable and
enforceable obligation, except when it is a part of the wage or salary
compensation" (Philippine Education Co. vs. CIR and the Union of
Philippine Co. Employees [NLU], 92 Phil. 381; Ansay, et. al. vs.
National Development Co., et. al., 107 Phil. 998, 999; Emphasis
supplied).

In Atok Big Wedge Mining Co. vs. Atok Big Wedge Mutual Benefit
Association (92 Phil. 754), this Court, thru Justice Labrador, held:
Whether or not bonus forms part of wages
depends upon the condition or circumstance for
its payment. If it is an additional compensation
WHICH THE EMPLOYER PROMISED AND
AGREED to give without any condition imposed
for its payment ... then it is part of the wage.
(Emphasis supplied).1wph1.t
In Altomonte vs. Philippine American Drug Co. (106 Phil. 137), the
Supreme Court held that an employee is not entitled to bonus where
there is no showing that it had been granted by the employer to its
employees periodically or regularly as to become part of their wages
or salaries. The clear implication is that bonus is recoverable as part
of the wage or salary where the employer regularly or periodically
gives it to employees.
American jurisprudence equally regards bonuses as part of
compensation or recoverable wages.
Thus, it was held that "... it follows that in determining the regular rate
of pay, a bonus which in fact constitutes PART OF AN EMPLOYEE'S
compensation, rather than a true gift or gratuity, has to be taken into
consideration." (48 Am. Jur. 2d, Labor and Labor Relations, No.
1555, citing the cases of Triple "AAA" Co. vs. Wirtz and Haber vs.
Americana Corporation; Emphasis supplied). It was further held that
"... the regular rate includes incentive bonuses paid to the employees
in addition to the guaranteed base rates regardless of any contract
provision to the contrary and even though such bonuses could not be
determined or paid until such time after the pay day" (48 Am. Jur. 2d,
Labor and Labor Relations, No. 1555, citing the case of Walling vs.
Harnischfeger Corp., 325 US 427, 89 L Ed 1711, 65 S Ct. 1246;
Emphasis supplied).1wph1.t
Petitioners in the present case do not dispute that as a matter of
tradition, the company has been doling out bonuses to employees. In
fact, the company balance sheets for the years 1956 to 1962
contained bonus and pension computations which were never
repudiated or questioned by petitioners. As such, bonus for a given
year earmarked as a matter of tradition for distribution to employees
has formed part of their recoverable wages from the company.
Moreover, with greater reason, should recovery of bonuses as part of
back wages be observed in the present case since the company, in
the light of the very admission of company accountant Francisco
Cusi, distributes bonuses to its employees even if the company has
suffered losses. Specifically, petitioner company has done this in
1962 (t.s.n., p. 149, Sept. 20, 1965).
Since bonuses are part of back wages of private respondents, the
order of May 30, 1969, directing the payment of their bonuses, did
not amend the decision of September 16, 1963 of respondent Court
directing payment of their wages, which has long become final and
executory, in the same way that the previous order of May 14, 1964
granting execution of said decision of September 16, 1963 also
directed the computation of the wages to be paid to private
respondents as decreed by the decision of September 16, 1963. All
the orders of May 30, 1969, November 28, 1966 and May 14, 1964
merely implement the already final and executory decision of
September 16, 1963.
Petitioners insist that We adopt the ruling in the Sta. Cecilia Sawmills
case wherein the recoverable back wages were limited to only three
(3) months; because as in the Sta. Cecilia Sawmills case, the
Claparols Steel and Nail Plant ceased operations due to enormous
business reverses.
Respondent Court's findings that indeed the Claparols Steel and Nail
Plant, which ceased operation of June 30, 1957, was SUCCEEDED
by the Claparols Steel Corporation effective the next day, July 1,
1957 up to December 7, 1962, when the latter finally ceased to
operate, were not disputed by petitioners. It is very clear that the
latter corporation was a continuation and successor of the first entity,
and its emergence was skillfully timed to avoid the financial liability
that already attached to its predecessor, the Claparols Steel and Nail
Plant. Both predecessors and successor were owned and controlled
by the petitioner Eduardo Claparols and there was no break in the
succession and continuity of the same business. This "avoiding-theliability" scheme is very patent, considering that 90% of the
subscribed shares of stocks of the Claparols Steel Corporation (the
second corporation) was owned by respondent (herein petitioner)
Claparols himself, and all the assets of the dissolved Claparols Steel
and Nail Plant were turned over to the emerging Claparols Steel
Corporation.

It is very obvious that the second corporation seeks the protective


shield of a corporate fiction whose veil in the present case could, and
should, be pierced as it was deliberately and maliciously designed to
evade its financial obligation to its employees.
It is well remembering that in Yutivo & Sons Hardware Company vs.
Court of Tax Appeals (L-13203, Jan. 28, 1961, 1 SCRA 160), We
held that when the notion of legal entity is used to defeat public
convenience, justify wrong, protect fraud, or defend crime, the law
will regard the corporation as an association or persons, or, in the
case of two corporations, will merge them into one.
In Liddel & Company, Inc. vs. Collector of Internal Revenue (L-9687,
June 30, 1961, 2 SCRA 632), this Court likewise held that where a
corporation is a dummy and serves no business purpose and is
intended only as a blind, the corporate fiction may be ignored.
In Commissioner of Internal Revenue vs. Norton and Harrison
Company (L-17618, Aug. 31, 1964, 11 SCRA 714), We ruled that
where a corporation is merely an adjunct, business conduit or alter
ego of another corporation, the fiction of separate and distinct
corporate entities should be disregarded.
To the same uniform effect are the decisions in the cases of
Republic vs. Razon (L-17462, May 29, 1967, 20 SCRA 234) and
A.D. Santos, Inc. vs. Vasquez (L-23586, March 20, 1968, 22 SCRA
1156).
WE agree with respondent Court of Industrial Relations, therefore,
that the amount of back wages recoverable by respondent workers
from petitioners should be the amount accruing up to December 7,
1962 when the Claparols Steel Corporation ceased operations.
Cruz vs Dalisay
Business Organization Corporation Law Piercing the Veil of Corporate
Fiction Exercised by the Wrong Person
In 1984, the National Labor Relations Commission issued an order against
Qualitrans Limousine Service, Inc. (QLSI) ordering the latter to reinstate
the employees it terminated and to pay them backwages. Quiterio Dalisay,
Deputy Sheriff of the court, to satisfy the backwages, then garnished the
bank account of Adelio Cruz. Dalisay justified his act by averring that Cruz
was the owner and president of QLSI. Further, he claimed that the counsel
for the discharged employees advised him to garnish the account of Cruz.
ISSUE: Whether or not the action of Dalisay is correct.
HELD: No. What Dalisay did is tantamount to piercing the veil of
corporate fiction. He actually usurped the power of the court. He also
overstepped his duty as a deputy sheriff. His duty is merely ministerial and
it is incumbent upon him to execute the decision of the court according to
its tenor and only against the persons obliged to comply. In this case, the
person judicially named to comply was QLSI and not Cruz. It is a wellsettled doctrine both in law and in equity that as a legal entity, a corporation
has a personality distinct and separate from its individual stockholders or
members. The mere fact that one is president of a corporation does not
render the property he owns or possesses the property of the corporation,
since the president, as individual, and the corporation are separate entities.

Cruz vs Dalisay
Administrative Matter in the Supreme Court.Malfeasance in office, corrupt
practices and serious irregularities.
Doctrine: A corporation has a personality distinct and separate from its
individual stockholders or members.Facts:
1.
In a sworn complaint dated July 23, 1984, Adelio Cruz (complainant)
charged Quiterio Dalisay (respondent),Senior Deputy Sh eriff of
Manila, with malfeasance in office, corrupt practices and
seri ous irregularities allegedly committed as follows:
a.
Respondent attached and/or levied the m oney b elonging t o
complainant Cruz wh en he was not himself the judgment debtor in
the final judgment of an NLRC case sought to be enforced but rather the
company known as Qualitrans Limousine Service,
Inc..b . R e s p o n d e n t a l s o c a u s e d t h e s e r v i c e o f t h e
alias writ of execution upon complainant who is
a resident of Pasa y Cit y, despite knowledge that his territorial
jurisdiction covers Manila only and does not extend to Pasay City.
2.
In his Comment, respondent explained that when he garnished
complainants cash deposit at the Philtrustbank he was merely performing a
ministerial duty. And that while it is true that said writ was addressed
toQualitrans Limousine Service, Inc., it is also a fact that
complainant had exec uted an affidavit before the Pasay City

assistant fiscal stating that he is the owner/ president of Qualitrans. Because


of that declaration,the counsel for the plaintiff in the labor case
advised him to serve notice of garnishment on the
Philtrust bank.3.On Novemb er 1 2, 1984 this case was referred
to the executive judge of the R TC of Manila for
investigation, report and recommendation. However, prior to the
termination of the proceedings, complainant executed anaffidavit of
desistance stating that he is no longer interested in prosecuting the case and
that there was justa misunderstanding between complainant and
respondent.4.On May 29, 1986, acting on respondents m otion
the executive judge issu ed an order rec omm ending the dismissal
of the case.
Issue: WON the complaint should be dismissed based on complainants
motion of desistance.Held: NOReason:
1. It has been held that desistance of complainant does not
preclud e the taking of disciplinary action
against respondent.2.Respondents actuation in enf orcing a
judgment against complainant wh o is not a judgment debtor
in thecase calls for disciplinary action. What is incumbent upon
respondent is to ensure that only the portion of adecision ordained or
decreed in the dispositive part should be the subject of the execution.
3.
The tenor of the NLRC judgment and the implementing writ is clear
enough. It directed Qualitrans LimousineService, inc., to reinstate the
discharged employees and pay them full backwages. Respondent,
however,c h o o s e t o p i e r c e t h e v e i l o f c o r p o r a t e
entity usurping a power belonging to the court and
a s s u m e d improvidently that since the complainant is the owner/president
of Qualitrans Limousine Service, Inc., theyare one and the same. It is a well
settled doctrine both in law and equity that as a legal entity, a
corporationhas a personality distinct and separate from its individual
stockholders or members.
4.
The mere fact that one is president of the corporation does not render the
property he owns or possessesthe propert y of the corporation, since
that president, as an individual, and the corporation are
separate entities.
Decision: ACCORDINGLY, we find Respondent
Deputy Sheriff Quiterio l. Dalisay NEGLIGENT in
t h e enforcement of the writ of execution in NLRC Case No. 8-12389-91,
and a fine equivalent to 3 months salaryis hereby imposed with a stern
warning that the commission of the same or similar offense in the future
willmerit a heavier penalty. Let a copy of the Resolution be filed in the
personal record of the respondent
Cruz vs Dalisay
Doctrine: Doctrine of Piercing the Veil of Corporate Entity
A corporation has a personality distinct and separate from its
individual stockholders or members.
Facts:
A sworn complaint dated July 23, 1984 was filed by Adelio Cruz
charging Quiterio Dalisay, Senior Deputy Sheriff of Manila, with
malfeasance in office, corrupt practices and serious irregularities
allegedly committed as follows:
a. Respondent attached and/or levied the money belonging to
complainant Cruz when he was not himself the judgment debtor in
the final judgment of an NLRC case sought to be enforced but rather
the company known as Qualitrans Limousine Service, Inc.; and
b. Respondent also caused the service of the alias writ of execution
upon complainant who is a resident of Pasay City, despite
knowledge that his territorial jurisdiction covers Manila only and does
not extend to Pasay City.
Respondent in his reply explained that when he garnished
complainants cash deposit at the Philtrust bank he was merely
performing a ministerial duty. And that while it is true that said writ
was addressed to Qualitrans Limousine Service, Inc., it is also a fact
that complainant had executed an affidavit before the Pasay City
assistant fiscal stating that he is the owner/ president of Qualitrans.
Because of that declaration, the counsel for the plaintiff in the labor
case advised him to serve notice of garnishment on the Philtrust
bank.
Issue: Whether or not the personal property of Cruz (complainant) is
properly levied or attached as owner of the corporation?

Held: NO
Respondents actuation in enforcing a judgment against complainant
who is not a judgment debtor in the case calls for disciplinary action.
What is incumbent upon respondent is to ensure that only the portion
of a decision ordained or decreed in the dispositive part should be
the subject of the execution. The tenor of the NLRC judgment and
the implementing writ is clear enough. It directed Qualitrans
Limousine Service, Inc. in its judgment and not the owner thereof.
Respondent, however, choose to pierce the veil of corporate
entity usurping a power belonging to the court and assumed
improvidently that since the complainant is the owner/president of
Qualitrans Limousine Service, Inc., they are one and the same. It is a
well settled doctrine both in law and equity that as a legal entity, a

corporation has a personality distinct and separate from its individual


stockholders or members.
The mere fact that one is president of the corporation does not
render the property he owns or possesses the property of the
corporation, since that president, as an individual, and the
corporation are separate entities.
Tan Boon Bee vs Jarencio
163 SCRA 205 Business Organization Corporation Law Piercing the
Veil of Corporate Fiction Alter Ego Case
In 1972, Anchor Supply Co. (ASC), through Tan Boon Bee, entered into a
contract of sale with Graphic Publishing Inc. (GPI) whereby ASC shall
deliver paper products to GPI. GPI paid a down payment but defaulted in
paying the rest despite demand from ASC. ASC sued GPI and ASC won.
To satisfy the indebtedness, the trial court, presided by Judge Hilarion
Jarencio, ordered that one of the printing machines of GPI be auctioned. But
before the auction can be had, Philippine American Drug Company
(PADCO) notified the sheriff that PADCO is the actual owner of said
printing machine. Notwithstanding, the sheriff still went on with the auction
sale where Tan Boon Bee was the highest bidder.
Later, PADCO filed with the same court a motion to nullify the sale on
execution. The trial court ruled in favor of PADCO and it nullified said
auction sale. Tan Boon Bee assailed the order of the trial court. Tan Boon
Bee averred that PADCO holds 50% of GPI; that the board of directors of
PADCO and GPI is the same; that the veil of corporate fiction should be
pierced based on the premises. PADCO on the other hand asserts ownership
over the said printing machine; that it is merely leasing it to GPI.
ISSUE: Whether or not the veil of corporate fiction should be pierced.
HELD: Yes. PADCO, as its name suggests, is a drug company not engaged
in the printing business. So it is dubious that it really owns the said printing
machine regardless of PADCOs title over it. Further, the printing machine,
as shown by evidence, has been in GPIs premises even before the date
when PADCO alleged that it acquired ownership thereof. Premises
considered, the veil of corporate fiction should be pierced; PADCO and GPI
should be considered as one. When a corporation is merely an adjunct,
business conduit or alter ego of another corporation the fiction of separate
and distinct corporation entities should be disregarded.

COMMISSIONER OF INTERNAL REVENUE, petitioner,


vs.
NORTON and HARRISON COMPANY, respondent.
Office of the Solicitor General for petitioner.
Pio Joven for respondent.
PAREDES, J.:
This is an appeal interposed by the Commissioner of Internal Revenue
against the following judgment of the Court of Tax Appeals:
IN VIEW OF THE FOREGOING, we find no legal basis to
support the assessment in question against petitioner. If at all,
the assessment should have been directed against JACKBILT,
the manufacturer. Accordingly, the decision appealed from is
reversed, and the surety bond filed to guarantee payment of said
assessment is ordered cancelled. No pronouncement as to costs.
Norton and Harrison is a corporation organized in 1911, (1) to buy and sell
at wholesale and retail, all kinds of goods, wares, and merchandise; (2) to
act as agents of manufacturers in the United States and foreign countries;
and (3) to carry on and conduct a general wholesale and retail mercantile
establishment in the Philippines. Jackbilt is, likewise, a corporation
organized on February 16, 1948 primarily for the purpose of making,
producing and manufacturing concrete blocks. Under date of July 27, 1948.
Norton and Jackbilt entered into an agreement whereby Norton was made
the sole and exclusive distributor of concrete blocks manufactured by
Jackbilt. Pursuant to this agreement, whenever an order for concrete blocks
was received by the Norton & Harrison Co. from a customer, the order was
transmitted to Jackbilt which delivered the merchandise direct to the
customer. Payment for the goods is, however, made to Norton, which in
turn pays Jackbilt the amount charged the customer less a certain amount, as
its compensation or profit. To exemplify the sales procedures adopted by
the Norton and Jackbilt, the following may be cited. In the case of the sale
of 420 pieces of concrete blocks to the American Builders on April 1, 1952,
the purchaser paid to Norton the sum of P189.00 the purchase price. Out of
this amount Norton paid Jackbilt P168.00, the difference obviously being its
compensation. As per records of Jackbilt, the transaction was considered a

sale to Norton. It was under this procedure that the sale of concrete blocks
manufactured by Jackbilt was conducted until May 1, 1953, when the
agency agreement was terminated and a management agreement between
the parties was entered into. The management agreement provided that
Norton would sell concrete blocks for Jackbilt, for a fixed monthly fee of
P2,000.00, which was later increased to P5,000.00.
During the existence of the distribution or agency agreement, or on June 10,
1949, Norton & Harrison acquired by purchase all the outstanding shares of
stock of Jackbilt. Apparently, due to this transaction, the Commissioner of
Internal Revenue, after conducting an investigation, assessed the respondent
Norton & Harrison for deficiency sales tax and surcharges in the amount of
P32,662.90, making as basis thereof the sales of Norton to the Public. In
other words, the Commissioner considered the sale of Norton to the public
as the original sale and not the transaction from Jackbilt. The period
covered by the assessment was from July 1, 1949 to May 31, 1953. As
Norton and Harrison did not conform with the assessment, the matter was
brought to the Court of Tax Appeals.
The Commissioner of Internal Revenue contends that since Jackbilt was
owned and controlled by Norton & Harrison, the corporate personality of
the former (Jackbilt) should be disregarded for sales tax purposes, and the
sale of Jackbilt blocks by petitioner to the public must be considered as the
original sales from which the sales tax should be computed. The Norton &
Harrison Company contended otherwise that is, the transaction subject to
tax is the sale from Jackbilt to Norton.
Wherefore, the parties respectfully pray that the foregoing stipulation of
facts be admitted and approved by this Honorable Court, without prejudice
to the parties adducing other evidence to prove their case not covered by
this stipulation of facts. 1wph1.t
The majority of the Tax Court, in relieving Norton & Harrison of liability
under the assessment, made the following observations:
The law applicable to the case is Section 186 of the National
Internal Revenue Code which imposes a percentage tax of 7%
on every original sale of goods, wares or merchandise, such tax
to be based on the gross selling price of such goods, wares or
merchandise. The term "original sale" has been defined as the
first sale by every manufacturer, producer or importer. (Sec. 5,
Com. Act No. 503.) Subsequent sales by persons other than the
manufacturer, producer or importer are not subject to the sales
tax.
If JACKBILT actually sold concrete blocks manufactured by it
to petitioner under the distributorship or agency agreement of
July 27, 1948, such sales constituted the original sales which are
taxable under Section 186 of the Revenue Code, while the sales
made to the public by petitioner are subsequent sales which are
not taxable. But it appears to us that there was no such sale by
JACKBILT to petitioner. Petitioner merely acted as agent for
JACKBILT in the marketing of its products. This is shown by
the fact that petitioner merely accepted orders from the public
for the purchase of JACKBILT blocks. The purchase orders
were transmitted to JACKBILT which delivered the blocks to
the purchaser directly. There was no instance in which the
blocks ordered by the purchasers were delivered to the
petitioner. Petitioner never purchased concrete blocks from
JACKBILT so that it never acquired ownership of such concrete
blocks. This being so, petitioner could not have sold JACKBILT
blocks for its own account. It did so merely as agent of
JACKBILT. The distributorship agreement of July 27, 1948, is
denominated by the parties themselves as an "agency for
marketing" JACKBILT products. ... .
xxx

xxx

xxx

Therefore, the taxable selling price of JACKBILT blocks under


the aforesaid agreement is the price charged to the public and not
the amount billed by JACKBILT to petitioner. The deficiency
sales tax should have been assessed against JACKBILT and not
against petitioner which merely acted as the former's agent.
xxx

xxx

xxx

The majority opinion is now before Us on appeal by the Commissioner of


Internal Revenue, on four (4) assigned errors, all of which pose the
following propositions: (1) whether the acquisition of all the stocks of the
Jackbilt by the Norton & Harrison Co., merged the two corporations into a
single corporation; (2) whether the basis of the computation of the
deficiency sales tax should be the sale of the blocks to the public and not to
Norton.
It has been settled that the ownership of all the stocks of a corporation by
another corporation does not necessarily breed an identity of corporate
interest between the two companies and be considered as a sufficient
ground for disregarding the distinct personalities (Liddell & Co., Inc. v.
Coll. of Int. Rev. L-9687, June 30, 1961). However, in the case at bar, we
find sufficient grounds to support the theory that the separate identities of
the two companies should be disregarded. Among these circumstances,
which we find not successfully refuted by appellee Norton are: (a) Norton
and Harrison owned all the outstanding stocks of Jackbilt; of the 15,000
authorized shares of Jackbilt on March 31, 1958, 14,993 shares belonged to
Norton and Harrison and one each to seven others; (b) Norton constituted
Jackbilt's board of directors in such a way as to enable it to actually direct
and manage the other's affairs by making the same officers of the board for
both companies. For instance, James E. Norton is the President, Treasurer,
Director and Stockholder of Norton. He also occupies the same positions in
Jackbilt corporation, the only change being, in the Jackbilt, he is merely a
nominal stockholder. The same is true with Mr. Jordan, F. M. Domingo,
Mr. Mantaring, Gilbert Golden and Gerardo Garcia, while they are merely
employees of the North they are Directors and nominal stockholders of the
Jackbilt (c) Norton financed the operations of the Jackbilt, and this is shown
by the fact that the loans obtained from the RFC and Bank of America were
used in the expansion program of Jackbilt, to pay advances for the purchase
of equipment, materials rations and salaries of employees of Jackbilt and
other sundry expenses. There was no limit to the advances given to Jackbilt
so much so that as of May 31, 1956, the unpaid advances amounted to
P757,652.45, which were not paid in cash by Jackbilt, but was offset by
shares of stock issued to Norton, the absolute and sole owner of Jackbilt; (d)
Norton treats Jackbilt employees as its own. Evidence shows that Norton
paid the salaries of Jackbilt employees and gave the same privileges as
Norton employees, an indication that Jackbilt employees were also Norton's
employees. Furthermore service rendered in any one of the two companies
were taken into account for purposes of promotion; (e) Compensation given
to board members of Jackbilt, indicate that Jackbilt is merely a department
of Norton. The income tax return of Norton for 1954 shows that as
President and Treasurer of Norton and Jackbilt, he received from Norton
P56,929.95, but received from Jackbilt the measly amount of P150.00, a
circumstance which points out that remuneration of purported officials of
Jackbilt are deemed included in the salaries they received from Norton. The
same is true in the case of Eduardo Garcia, an employee of Norton but a
member of the Board of Jackbilt. His Income tax return for 1956 reveals
that he received from Norton in salaries and bonuses P4,220.00, but
received from Jackbilt, by way of entertainment, representation, travelling
and transportation allowances P3,000.00. However, in the withholding
statement (Exh. 28-A), it was shown that the total of P4,200.00 and
P3,000.00 (P7,220.00) was received by Garcia from Norton, thus portraying
the oneness of the two companies. The Income Tax Returns of Albert
Golden and Dioscoro Ramos both employees of Norton but board members
of Jackbilt, also disclose the game method of payment of compensation and
allowances. The offices of Norton and Jackbilt are located in the same
compound. Payments were effected by Norton of accounts for Jackbilt and
vice versa. Payments were also made to Norton of accounts due or payable
to Jackbilt and vice versa.
Norton and Harrison, while not denying the presence of the set up stated
above, tried to explain that the control over the affairs of Jackbilt was not
made in order to evade payment of taxes; that the loans obtained by it which
were given to Jackbilt, were necessary for the expansion of its business in
the manufacture of concrete blocks, which would ultimately benefit both
corporations; that the transactions and practices just mentioned, are not
unusual and extraordinary, but pursued in the regular course of business and
trade; that there could be no confusion in the present set up of the two
corporations, because they have separate Boards, their cash assets are
entirely and strictly separate; cashiers and official receipts and bank
accounts are distinct and different; they have separate income tax returns,
separate balance sheets and profit and loss statements. These explanations
notwithstanding an over-all appraisal of the circumstances presented by the
facts of the case, yields to the conclusion that the Jackbilt is merely an
adjunct, business conduit or alter ego, of Norton and Harrison and that the
fiction of corporate entities, separate and distinct from each, should be
disregarded. This is a case where the doctrine of piercing the veil of
corporate fiction, should be made to apply. In the case of Liddell & Co. Inc.
v. Coll. of Int. Rev., supra, it was held:

Presiding Judge Nable of the same Court expressed a partial dissent, stating:
Upon the aforestated circumstances, which disclose Norton's
control over and direction of Jackbilt's affairs, the corporate
personality of Jackbilt should be disregarded, and the
transactions between these two corporations relative to the
concrete blocks should be ignored in determining the percentage
tax for which Norton is liable. Consequently, the percentage tax
should be computed on the basis of the sales of Jackbilt blocks
to the public.

There are quite a series of conspicuous circumstances that


militates against the separate and distinct personality of Liddell
Motors Inc., from Liddell & Co. We notice that the bulk of the
business of Liddell & Co. was channel Red through Liddell
Motors, Inc. On the other hand, Liddell Motors Inc. pursued no
activities except to secure cars, trucks, and spare parts from
Liddell & Co., Inc. and then sell them to the general public.
These sales of vehicles by Liddell & Co, to Liddell Motors. Inc.
for the most part were shown to have taken place on the same

day that Liddell Motors, Inc. sold such vehicles to the public.
We may even say that the cars and trucks merely touched the
hands of Liddell Motors, Inc. as a matter of formality.
xxx

xxx

xxx

Accordingly, the mere fact that Liddell & Co. and Liddell
Motors, Inc. are corporations owned and controlled by Frank
Liddell directly or indirectly is not by itself sufficient to justify
the disregard of the separate corporate identity of one from the
other. There is however, in this instant case, a peculiar sequence
of the organization and activities of Liddell Motors, Inc.
As opined in the case of Gregory v. Helvering "the legal right of
a tax payer to decrease the amount of what otherwise would be
his taxes, or altogether avoid them, by means which the law
permits, cannot be doubted". But as held in another case, "where
a corporation is a dummy, is unreal or a sham and serves no
business purpose and is intended only as a blind, the corporate
form may be ignored for the law cannot countenance a form that
is bald and a mischievous fictions".
... a taxpayer may gain advantage of doing business thru a
corporation if he pleases, but the revenue officers in proper
cases, may disregard the separate corporate entity where it serves
but as a shield for tax evasion and treat the person who actually
may take benefits of the transactions as the person accordingly
taxable.

PHIVIDEC. On January 21, 1980, Borres filed a complaint for


damages against PRI and Panay Railways Inc. (Panay ), 2
whereupon the latter filed with leave of court a third-party complaint
against the herein petitioner. 3 It alleged that upon the sale to
PHILSUCOM of PRI, the corporate name of PRI was changed to
Panay Railways, Inc. It disclaimed liability on the ground that in the
Agreement concluded between PHIVIDEC and PHILSUCOM, it was
provided that:
D. With the exception of the Liabilities and
Contracts specified in Annexes 4 and 5 of the
preceding paragraph, PHIVIDEC hereby holds
PHILSUCOM harmless from and against any
action, claim or liability that may arise out of or
result from acts or omissions, contracts or
transactions prior to the turn-over.
After trial, Judge Ricardo M. Ilarde of the Regional Trial Court of Iloilo
held Phividec Railways, Inc. negligent and so liable to the plaintiff for
damages. It also held that as PRI was a wholly-owned subsidiary of
PHIVIDEC, the latter should answer for PRI's liability. The decision
was affirmed on appeal by the respondent court, 4 which is now
faulted for grave abuse of discretion in this petition.
The sole issue raised in this petition is the ruling of the Court of
Appeals that:
Thus, the piercing of the veil of corporate fiction
is called for in the case at bar. When PRI was
sold by PHIVIDEC to PHILSUCOM on May 25,
1979, the legal fiction of PRI as a separate
corporate entity from PHIVIDEC disappeared
pursuant to and in view of the representations
and warranties contained in the agreement of
sale between PHIVIDEC and PHILSUCOM,
particularly the stipulation already quoted above,
by virtue of which PHIVIDEC held PHILSUCOM
harmless from any claim or liability arising out of
any act or transaction "prior to the turn-over." By
virtue of this provision, PHIVIDEC had expressly
assumed liability for any claim arising before the
turn-over of PRI to PHILSUCOM. And since the
accident in question took place before said turnover and since after said turn-over PRI ceased
to exist (in the sense that its railways operations
were taken over by PHILSUCOM thru the Panay
RW) the only logical conclusion is that
PHIVIDEC should be solely liable for the
damages to the plaintiff in the case at bar.
Indeed, applying the Koppel precedent just
cited, PHIVIDEC cannot hide behind the veil of
corporate fiction in order to evade this liability,
nor could the veil of corporate fiction be made a
shield to confuse claimants such as plaintiffappellee.

... to allow a taxpayer to deny tax liability on the ground that the
sales were made through another and distinct corporation when
it is proved that the latter is virtually owned by the former or that
they are practically one and the same is to sanction a
circumvention of our tax laws. (and cases cited therein.)
In the case of Yutivo Sons Hardware Co. v. Court of Tax Appeals, L-13203,
Jan. 28, 1961, this Court made a similar ruling where the circumstances of
unity of corporate identities have been shown and which are identical to
those obtaining in the case under consideration. Therein, this Court said:
We are, however, inclined to agree with the court below that SM
was actually owned and controlled by petitioner as to make it a
mere subsidiary or branch of the latter created for the purpose of
selling the vehicles at retail (here concrete blocks) ... .
It may not be amiss to state in this connection, the advantages to Norton in
maintaining a semblance of separate entities. If the income of Norton
should be considered separate from the income of Jackbilt, then each would
declare such earning separately for income tax purposes and thus pay lesser
income tax. The combined taxable Norton-Jackbilt income would subject
Norton to a higher tax. Based upon the 1954-1955 income tax return of
Norton and Jackbilt (Exhs. 7 & 8), and assuming that both of them are
operating on the same fiscal basis and their returns are accurate, we would
have the following result: Jackbilt declared a taxable net income of
P161,202.31 in which the income tax due was computed at P37,137.00
(Exh. 8); whereas Norton declared as taxable, a net income of P120,101.59,
on which the income tax due was computed at P25,628.00. The total of
these liabilities is P50,764.84. On the other hand, if the net taxable earnings
of both corporations are combined, during the same taxable year, the tax
due on their total which is P281,303.90 would be P70,764.00. So that, even
on the question of income tax alone, it would be to the advantages of
Norton that the corporations should be regarded as separate entities.
WHEREFORE, the decision appealed from should be as it is hereby
reversed and another entered making the appellee Norton & Harrison liable
for the deficiency sales taxes assessed against it by the appellant
Commissioner of Internal Revenue, plus 25% surcharge thereon. Costs
against appellee Norton & Harrison.

Phil Vet Inv Devt Co vs CA


The concept of piercing the veil of corporate fiction is a mystique to
many people, especially the layman. But it is not as esoteric as all
that as this case will demonstrate.
This case arose when Violeta M. Borres, private respondent herein,
was injured in an accident that was later held by the trial and
respondent courts to be due to the negligence of Phividec Railways,
Inc. (PRI). 1 The accident occurred on March 29, 1979. On May 25,
1979, petitioner Philippine Veterans Investment Development
Corporation (PHIVIDEC) sold all its rights and interests in the PRI to
the Philippine Sugar Commission (PHILSUCOM). Two days later,
PHILSUCOM caused the creation of a wholly-owned subsidiary, the
Panay Railways, Inc., to operate the railway assets acquired from

It is the position of the petitioner that PHIVIDEC and PRI are entirely
distinct and separate corporations although the latter is its
subsidiary. The transfer of the shares of stock of PRI to
PHILSUCOM did not divest PRI of its juridical personality or of its
capacity to direct its own affairs and conduct its own business under
the control of its own board of directors. By the same token, it is
answerable for its own obligations, which cannot be passed on to the
petitioner as its own liability. To support this stand, the petitioner
invokes the case of E.J. Nell v. Pacific Farms, 5 which, however, it
has not accurately quoted.
We must sustain the respondents.
In Koppel v. Yatco, 6 the Court, citing Fletcher, declared that the veil
of corporate fiction may be pierced when it is used to defeat public
convenience, justify wrong, protect fraud, or defend crime. 7 It added
that when the corporation is the mere alter ego or business conduit
of a person it may be disregarded, "to prevent injustice, or the
distortion or hiding of the truth, or to let in a just defense." 8
The rule is that:
Where it appears that two business enterprises
are owned, conducted and controlled by the
same parties, both law and equity will, when
necessary to protect the rights of third persons,
disregard the legal fiction that two corporations
are distinct entities, and treat them as identical. 9
In Yutivo Sons Hardware Co. v. Court of Tax Appeals, 10 this Court
held:

It is an elementary and fundamental principle of


corporation law that a corporation is an entity
separate and distinct from its stockholders and
from other corporations to which it may be
connected. However, "when the notion of legal
entity is used to defeat public convenience,
justify wrong, protect fraud or defend crime," the
law will regard the corporation as an association
of persons, or in the case of two corporations
merge them into one. ... Another rule is that,
when the corporation is the "mere alter ego or
business conduit of a person, it may be
disregarded."
In Commissioner of Internal Revenue v. Norton and Harrison Co., 11
this Court likewise ruled that where a corporation is merely an
adjunct, business conduit or alter ego of another corporation the
fiction of separate and distinct corporate entities should be
disregarded.
In fact, contrary to the suggestion in the petition, what the Court said
in the Nell Case was:
Generally where one corporation sells or
otherwise transfers all of its assets to another
corporation, the latter is not liable for the debts
and liabilities of the transferor, except: (1) where
the purchaser expressly or impliedly agrees to
assume such debts; (2) where the transaction
amounts to a consolidation or merger of the
corporations; (3) where the purchasing
corporation is merely a continuation of the
selling corporation; and (4) where the
transaction is entered into fraudulently in order
to escape liability for such debts.
Moreover, as correctly pointed out by the respondent court:
Besides, PHIVIDEC'S act of selling PRI to
PHILSUCOM shows that PHVIDEC had
complete control of PRI's business. This
circumstance renders applicable the rule cited
by third-party plaintiff-appellee (Costan v. Manila
Electric, 24 F 2nd 383) that if a parent- holding
company (PHIVIDEC in the present case)
assumes complete control of the operations of
its subsidiary's business, the separate corporate
existence of the subsidiary must be disregarded,
such that the holding company will be
responsible for the negligence of the employees
of the subsidiary as if it were the holding
company's own employees.
It is clear from the evidence of record that by virtue of the agreement
between PHIVIDEC and PHILSUCOM, particularly the stipulation
exempting the latter from any "claim or liability arising out of any act
or transaction" prior to the turn-over, PHIVIDEC had expressly
assumed liability for any claim against PRI. Since the accident
happened before that agreement and PRI ceased to exist after the
turn-over, it should follow that PHIVIDEC cannot evade its liability for
the injuries sustained by the private respondent.
A contrary conclusion would leave the private respondent without
any recourse for her legitimate claim. In the interest of justice and
equity, and to prevent the veil of corporate fiction from denying her
the reparation to which she is entitled, that veil must be pierced and
PHIVIDEC and PRI regarded as one and the same entity.
WHEREFORE, the challenged decision is AFFIRMED and the
petition is DENIED, with costs against the petitioner. It is so ordered.

Telephone Engineering and Service Co vs Wormens Comppensation


These certiorari proceedings stem from the award rendered against
petitioner Telephone Engineering and Services, Co., Inc. (TESCO)
on October 6, 1967 by the Acting Referee of Regional Office No. 4,
Quezon City Sub-Regional Office, Workmen's Compensation
Section, in favor of respondent Leonila S. Gatus and her children,
dependents of the deceased employee Pacifico L. Gatus. The
principal contention is that the award was rendered without
jurisdiction as there was no employer-employee relationship between
petitioner and the deceased.
Petitioner is a domestic corporation engaged in the business of
manufacturing telephone equipment with offices at Sheridan Street,

Mandaluyong, Rizal. Its Executive Vice-President and General


Manager is Jose Luis Santiago. It has a sister company, the Utilities
Management Corporation (UMACOR), with offices in the same
location. UMACOR is also under the management of Jose Luis
Santiago.
On September 8, 1964, UMACOR employed the late Pacifica L.
Gatus as Purchasing Agent. On May 16, 1965, Pacifico L. Gatus was
detailed with petitioner company. He reported back to UMACOR on
August 1, 1965. On January 13, 1967, he contracted illness and
although he retained to work on May 10, 1967, he died nevertheless
on July 14, 1967 of "liver cirrhosis with malignant degeneration."
On August 7, 1967, his widow, respondent Leonila S. Gatus, filed a
"Notice and Claim for Compensation" with Regional Office No. 4,
Quezon City Sub-Regional Office, Workmen's Compensation
Section, alleging therein that her deceased husband was an
employee of TESCO, and that he died of liver cirrhosis. 1 On August
9, 1967, and Office wrote petitioner transmitting the Notice and for
Compensation, and requiring it to submit an Employer's Report of
Accident or Sickness pursuant to Section 37 of the Workmen's
Compensation Act (Act No. 3428). 2 An "Employer's Report of
Accident or Sickness" was thus submitted with UMACOR indicated
as the employer of the deceased. The Report was signed by Jose
Luis Santiago. In answer to questions Nos. 8 and 17, the employer
stated that it would not controvert the claim for compensation, and
admitted that the deceased employee contracted illness "in regular
occupation." 3 On the basis of this Report, the Acting Referee
awarded death benefits in the amount of P5,759.52 plus burial
expenses of P200.00 in favor of the heirs of Gatus in a letter-award
dated October 6, 1967 4 against TESCO.
Replying on October 27, 1967, TESCO, through Jose Luis Santiago,
informed the Acting Referee that it would avail of the 15-days-notice
given to it to state its non-conformity to the award and contended
that the cause of the illness contracted by Gatus was in no way
aggravated by the nature of his work. 5
On November 6, 1967, TESCO requested for an extension of ten
days within which to file a Motion for Reconsideration, 6 and on
November 15, 1967, asked for an additional extension of five days. 7
TESCO filed its "Motion for Reconsideration and/or Petition to Set
Aside Award" on November 18, 1967, alleging as grounds therefor,
that the admission made in the "Employer's Report of Accident or
Sickness" was due to honest mistake and/or excusable negligence
on its part, and that the illness for which compensation is sought is
not an occupational disease, hence, not compensable under the law.
8
The extension requested was denied. The Motion for
Reconsideration was likewise denied in an Order issued by the Chief
of Section of the Regional Office dated December 28, 1967 9
predicated on two grounds: that the alleged mistake or negligence
was not excusable, and that the basis of the award was not the
theory of direct causation alone but also on that of aggravation. On
January 28, 1968, an Order of execution was issued by the same
Office.
On February 3, 1968, petitioner filed an "Urgent Motion to Compel
Referee to Elevate the Records to the Workmen's Compensation
Commission for Review." 10 Meanwhile, the Provincial Sheriff of Rizal
levied on and attached the properties of TESCO on February 17,
1968, and scheduled the sale of the same at public auction on
February 26, 1968. On February 28, 1968, the Commission issued
an Order requiring petitioner to submit verified or true copies of the
Motion for Reconsideration and/or Petition to Set Aside Award and
Order of December 28, 1967, and to show proof that said Motion for
Reconsideration was filed within the reglementary period, with the
warning that failure to comply would result in the dismissal of the
Motion. However, before this Order could be released, TESCO filed
with this Court, on February 22, 1968, The present petition for
"Certiorari with Preliminary Injunction" seeking to annul the award
and to enjoin the Sheriff from levying and selling its properties at
public auction.
On February 29, 1968, this Court required respondents to answer
the Petition but denied Injunction. 11 TESCO'S Urgent Motion dated
April 2, 1968, for the issuance of a temporary restraining order to
enjoin the Sheriff from proceeding with the auction sale of its
properties was denied in our Resolution dated May 8, 1968.
TESCO asserts: 1wph1.t
I. That the respondent Workmen's
Compensation Commission has no jurisdiction
nor authority to render the award (Annex 'D',
Petition) against your petitioner there being no
employer-employee relationship between it and
the deceased Gatus;

II. That petitioner can never be estopped from


questioning the jurisdiction of respondent
commission especially considering that
jurisdiction is never conferred by the acts or
omission of the parties;

Although this rule admits of exceptions, as where public welfare and


the advancement of public policy so dictate, the broader interests of
justice so require, or where the Orders complained of were found to
be completely null and void or that the appeal was not considered
the appropriate remedy, 23 the case at bar does not fan within any of
these exceptions. WHEREFORE, this Petition is hereby dismissed.

III. That this Honorable Court has jurisdiction to


nullify the award of respondent commission.
Alhambra Cigar vs SEC
TESCO takes the position that the Commission has no jurisdiction to
render a valid award in this suit as there was no employer-employee
relationship between them, the deceased having been an employee
of UMACOR and not of TESCO. In support of this contention,
petitioner submitted photostat copies of the payroll of UMACOR for
the periods May 16-31, 1967 and June 1-15, 1967 12 showing the
name of the deceased as one of the three employees listed under
the Purchasing Department of UMACOR. It also presented a
photostat copy of a check of UMACOR payable to the deceased
representing his salary for the period June 14 to July 13, 1967. 13
Both public and private respondents contend, on the other hand, that
TESCO is estopped from claiming lack of employer employee
relationship.
To start with, a few basic principles should be re-stated the existence
of employer-employee relationship is the jurisdictional foundation for
recovery of compensation under the Workmen's Compensation Law.
14
The lack of employer-employee relationship, however, is a matter
of defense that the employer should properly raise in the
proceedings below. The determination of this relationship involves a
finding of fact, which is conclusive and binding and not subject to
review by this Court. 15
Viewed in the light of these criteria, we note that it is only in this
Petition before us that petitioner denied, for the first time, the
employer-employee relationship. In fact, in its letter dated October
27, 1967 to the Acting Referee, in its request for extension of time to
file Motion for Reconsideration, in its "Motion for Reconsideration
and/or Petition to Set Aside Award," and in its "Urgent Motion to
Compel the Referee to Elevate Records to the Commission for
Review," petitioner represented and defended itself as the employer
of the deceased. Nowhere in said documents did it allege that it was
not the employer. Petitioner even admitted that TESCO and
UMACOR are sister companies operating under one single
management and housed in the same building. Although respect for
the corporate personality as such, is the general rule, there are
exceptions. In appropriate cases, the veil of corporate fiction may be
pierced as when the same is made as a shield to confuse the
legitimate issues. 16
While, indeed, jurisdiction cannot be conferred by acts or omission of
the parties, TESCO'S denial at this stage that it is the employer of
the deceased is obviously an afterthought, a devise to defeat the law
and evade its obligations. 17 This denial also constitutes a change of
theory on appeal which is not allowed in this jurisdiction. 18 Moreover,
issues not raised before the Workmen's Compensation Commission
cannot be raised for the first time on appeal. 19 For that matter, a
factual question may not be raised for the first time on appeal to the
Supreme Court. 20
This certiorari proceeding must also be held to have been
prematurely brought. Before a petition for certiorari can be instituted,
all remedies available in the trial Court must be exhausted first. 21
certiorari cannot be resorted to when the remedy of appeal is
present. 22 What is sought to be annulled is the award made by the
Referee. However, TESCO did not pursue the remedies available to
it under Rules 23, 24 and 25 of the Rules of the Workmen's
Compensation Commission, namely, an appeal from the award of
the Referee, within fifteen days from notice, to the Commission; a
petition for reconsideration of the latter's resolution, if adverse, to the
Commission en banc; and within ten days from receipt of an
unfavorable decision by the latter, an appeal to this Court. As
petitioner had not utilized these remedies available to it, certiorari win
not he, it being prematurely filed. As this Court ruled in the case of
Manila Jockey Club, Inc. vs. Del Rosario, 2 SCRA 462 (1961).
1wph1.t
An aggrieved party by the decision of a
Commissioner should seek a reconsideration of
the decision by the Commission en banc. If the
decision is adverse to him, he may appeal to the
Supreme Court. An appeal brought to the
Supreme Court without first resorting to the
remedy referred to is premature and may be
dismissed.

Business Organization Corporation Law Corporate Lifespan


On January 15, 1912, Alhambra Cigar & Cigarette Manufacturing
Company, Inc. was incorporated. Its lifespan was for 50 years so on January
15, 1962, it expired. Thereafter, its Board authorized its liquidation. Under
the prevailing law, Alhambra has 3 years to liquidate.
In 1963, while Alhambra was liquidating, Republic Act 3531 was enacted.
It amended Section 18 of the Corporation Law; it empowered domestic
private corporations to extend their corporate life beyond the period fixed
by the articles of incorporation for a term not to exceed fifty years in any
one instance. Previous to Republic Act 3531, the maximum non-extendible
term of such corporations was fifty years.
Alhambra now amended its articles of incorporation to extend its lifespan
for another 50 years. The Securities and Exchange Commission (SEC)
denied the amended articles of incorporation.
ISSUE: Whether or not a corporation under liquidation may still amend its
articles of incorporation to extend its lifespan.
HELD: No. Alhambra cannot avail of the new law because it has already
expired at the time of its passage. When a corporation is liquidating
pursuant to the statutory period of three years to liquidate, it is only
allowed to continue for the purpose of final closure of its business and no
other purposes. In fact, within that period, the corporation is enjoined from
continuing the business for which it was established. Hence, Alhambras
board cannot validly amend its articles of incorporation to extend its
lifespan.

Alhambra vs Cigar
Alhambra Cigar and Cigarette Manufacturing
C o m p a n y , I n c . w a s d u l y incorporated under Philippine laws on
its corporate articles it was to e#ist for fifty $5%&
years from incorporation. Its term of e#istence e#pired on
(n that date, it ceased transacting business, entered into a state of li)uidation.
*hereafter, a new corporation, Alhambra Industries, Inc., was formed to
+5+1 was enacted into la w. It amended /ection 10 of the
rations
to e#tend their corporate life beyond the period fi#ed by the articles of
incorporation for a term not to e# ceed fift y yea rs in any one
-e#tendible
term of such corporations was fift y yea rs. (n July 15,
r e s o l 2 e d t o a m e n d paragraph 34ourth3 of its articles of
incorporation to e#tend its corporate life for an a d d i t i o n a l f i f t y
years, or a total of 1%% years from its
incorporation.
articles of incorporation as so amended
c e r t i f i e d c o r r e c t b y i t s p r e s i d e n t a n d secretary and a
/6C, howe2 er, return ed said amended articles of incorporation

, cannot be a2ailed of b y the s aid corporation, for the reason


that its term of e#ist ence had already e#pired when the said law too7
effect in short, said law has no retroacti2e effect.3
8hether or not a corporation can e#tend its life by
amendment of its a r t i c l e s o f i n c o r p o r a t i o n
effected during the three-year statutory
p e r i o d f o r li)uidation when its original term of e#istence had already
e#pired.
Plain from the language of the pro2isi
is its meaning: continuance of a 3dissol2ed3 corporation as a
body c orporate for t h r e e y e a r s h a s f o r i t s p u r p o s e t h e
final closure of its affairs,
and no other;
the
from 3continuing the
business for which it was established3. *he li)uidation of the
became necessary

precisely because its life had ended. 4or this reason alone, the corporate
ty of that corporation to do business may
no lon ger be e#tended. And it should be clearly e2ident that
no corporation in a s t a t e o f l i ) u i d a t i o n c a n a c t i n a n y
way, much less
amend its articles
, 3 f o r t h e purpose of continuing the business for which it was
established
Narra Nickel Mining Corp vs Redmont Consolidated Mining
Corporations; nationality. The control test is still the prevailing mode of
determining whether or not a corporation is a Filipino corporation, within
the ambit of Sec. 2, Art. II of the 1987 Constitution, entitled to undertake
the exploration, development and utilization of the natural resources of
the Philippines. When in the mind of the Court there is doubt, based on
the attendant facts and circumstances of the case, in the 60-40 Filipinoequity ownership in the corporation, then it may apply the grandfather
rule.

NARRA NICKEL MINING AND DEVELOPMENT CORP., et al. v.


REDMONT CONSOLIDATED MINES CORP., G.R. No. 195580,
April 21, 2014
Remedial law; When is a case deemed moot and academic. A case is said to
be moot and/or academic when it ceases to present a justiciable
controversy by virtue of supervening events, so that a declaration thereon
would be of no practical use or value. Thus, the courts generally decline
jurisdiction over the case or dismiss it on the ground of mootness.
Exceptions to assuming jurisdiction despite the case becoming moot. The
mootness principle, however, does accept certain exceptions and the mere
raising of an issue of mootness will not deter the courts from trying a case
when there is a valid reason to do so. In David v. Macapagal-Arroyo, the
Court provided four instances where courts can decide an otherwise moot
case, thus: (1) There is a grave violation of the Constitution; (2) The
exceptional character of the situation and paramount public interest is
involved; (3) When constitutional issue raised requires formulation of
controlling principles to guide the bench, the bar, and the public; and (4)
The case is capable of repetition yet evading review.
Commercial law; Tests to determine the nationality of a corporation. There
are two acknowledged tests in determining the nationality of a corporation:
the control test and the grandfather rule. Paragraph 7 of DOJ Opinion No.
020, Series of 2005, adopts the 1967 SEC Rules which implemented the
requirement of the Constitution and other laws pertaining to the controlling
interests in enterprises engaged in the exploitation of natural resources
owned by Filipino citizens. The first part of paragraph 7, DOJ Opinion No.
020, stating shares belonging to corporations or partnerships at least 60%
of the capital of which is owned by Filipino citizens shall be considered as
of Philippine nationality, pertains to the control test or the liberal rule. On
the other hand, the second part of the DOJ Opinion which provides, if the
percentage of the Filipino ownership in the corporation or partnership is less
than 60%, only the number of shares corresponding to such percentage shall
be counted as Philippine nationality, pertains to the stricter, more stringent
grandfather rule.
Application of the Grandfather Rule. Based on the said SEC Rule and DOJ
Opinion, the Grandfather Rule or the second part of the SEC Rule applies
only when the 60-40 Filipino-foreign equity ownership is in doubt (i.e., in
cases where the joint venture corporation with Filipino and foreign
stockholders with less than 60% Filipino stockholdings [or 59%] invests in
other joint venture corporation which is either 60-40% Filipino-alien or the
59% less Filipino). Stated differently, where the 60-40 Filipino- foreign
equity ownership is not in doubt, the Grandfather Rule will not apply.
Existence of doubt. The assertion of petitioners that doubt only exists
when the stockholdings are less than 60% fails to convince this Court. DOJ
Opinion No. 20, which petitioners quoted in their petition, only made an
example of an instance where doubt as to the ownership of the
corporation exists. It would be ludicrous to limit the application of the said
word only to the instances where the stockholdings of non-Filipino
stockholders are more than 40% of the total stockholdings in a corporation.
The corporations interested in circumventing our laws would clearly strive
to have 60% Filipino Ownership at face value. It would be senseless for
these applying corporations to state in their respective articles of
incorporation that they have less than 60% Filipino stockholders since the
applications will be denied instantly. Thus, various corporate schemes and
layerings are utilized to circumvent the application of the Constitution.

Вам также может понравиться